Anatomy Final Questions

Réussis tes devoirs et examens dès maintenant avec Quizwiz!

According to Moore, the muscles that act to extend the femur at the hip and flex the knee are innervated by: a. Tibial division of sciatic b. Fibular division of sciatic c. Femoral d. Obturator e. More than one of the above is correct

A

Action(s) of the splenius muscles includes: a) Bilateral cervical extension b) Unilateral lateral flexion and rotation of the head to the opposite side c) Bilateral cervical flexion d) Bilateral cervical rotation

A

Afferent (A) and Efferent (E) means information is conveyed? a) A=to the CNS, E=from the CNS b) A=from the CNS, E=to the CNS c) A and E=to the CNS d) WHAT?

A

After several years of successful practice as a DPT, hand therapist, Mackenzie continues on to medical school becoming a hand surgeon. Dr. Borrowman confirms her diagnosis of Nathan's hand weakness by asking the patient to: a) Abduct and adduct his fingers b) Abduct his hand at his wrist c) Flex his elbow d) Abduct his arm

A

All of the muscles of the superficial posterior compartment have this common attachment: a) Calcaneal tuberosity via Achilles tendon b) Navicular tuberosity c) Styloid process of the fifth metatarsal d) Dome of the talus

A

Another name for the vertebral border of the scapula is the: a) Medial border b) Lateral border c) Superior border d) Inferior border

A

As Riley and Haley dissect through the muscle layers of the shoulder and back between the scapulae they proceed with extreme caution. In order from the most superficial to deep muscle layers Haley says that they will define: a) Trapezius, rhomboids, serratus posterior superior, longissimus thoracis b) Rhomboids, serratus posterior superior, longissimus thoracis c) Longissimus thoracis, trapezius, rhomboids D) Serratus posterior superior, trapezius, rhomboids, longissimus thoracis

A

Brittani, DPT is working on the respiratory floor. He sees a youngster that has somehow inhaled a small toy soldier into one of his main bronchii. Brittani knows which bronchus the toy is lodged in as: a) The right main bronchus is more vertical and has a larger diameter b) The left main bronchus is more vertical and has a larger diameter c) The right is shorter and branches off of the trachea more superiorly d) The left is shorter and branches off of the trachea more superiorly

A

Chelsea says: the muscle that does NOT have a scapular attachment is the: a) Serratus posterior superior b) Pectoralis minor c) Serratus anterior d) Trapezius

A

Chelsea, DPTS sees a left handed patient who was a passenger in an automobile accident and sustained a midhumeral fracture and a subsequent injury to the radial nerve of his left arm. She learns from the post operative report that all of the muscles innervated by the radial nerve distal to the injury site are temporarily paralyzed. Muscles involved would include the: a) Brachioradialis and extensor carpi radialis longus b) Abductor pollicis longus and adductor pollicis c) Abductor pollicis longus and abductor pollicis brevis d) Flexor carpi radialis and extensor carpi radialis

A

Contrecoup skull injuries: a. Occur at a point on the brain opposite the point of impact b. Always involve the medulla oblongata c. Are those involving only the lacrimal bone d. Never involve the temporal bone e. More than one of the above are correct

A

Dermatomal testing of C2 involves lightly stroking what region of the patient? a) Posterior occiput b) Anterior neck c) Supraclavicular fossa d) Lateral arm over deltoid

A

Dermatomes are part of? a) PNS and somatic b) CNS and somatic c) PNS and autonomic d) CNS and autonomic

A

Diastasis recti is best described as a) A separation of the left and right rectus abdominis muscles b) Horizontal regions of the rectus abdominis muscles that give a six-pack appearance on a lean athlete c) The membranous layer of fascia deep to the transverse abdominis muscles d) A line above which all of the layers of the rectus sheath pass posteriorly to the rectus abdominis muscles

A

Dr Folger plays tennis all week and develops 'tennis elbow'. Riley Beck, DPTS, conducts the evaluation and asks "where does it hurt?". Dr. Folger is most likely to complain of pain: a) Over the lateral epicondyle of the humerus b) Over the medial epicondyle of the humerus c) Over the cubital fossa d) Over the olecranon fossa

A

Dr. Nunn, DPT tells us that a patient with Legg-Calve-Perthes Disease has: a. Femoral head that is displaced and flattened b. Chronic cramping of the calf muscles c. A fracture at the epiphysis between the femoral head and neck d. Lost circulation to the condyles of the femur e. More than one of the above

A

During Clinical Practicum III you are evaluating a patient whose diagnosis is cerebral vascular accident (CVA/stroke), your assessment reveals the patient having significant difficulty speaking. What artery occlusion/infarct led to the damage of the brain that controls the ability to speak (Broca's areal/lateral portion of frontal lobe)? a. Middle Cerebral Artery b. Posterior Cerebral Artery c. Middle Meningeal Artery d. External Carotid Artery

A

Either "To Zanzibar By Motor Car" or "Tiny Zebras Bite My Cheek" works as a pneumonic to describe the branches of CN? a. VII b. II c. IV d. V

A

Flexors of the leg at the knee include? a. All 4 Hamstrings b. Adductor longus c. Pectineus d. More than one of the above are correct!

A

Forceful eversion and dorsiflexion affect which ligament? a) Deltoid b) Spring c) ATF d) PTF

A

Fusion of the diaphysis with the epiphysis: a) Occurs at different ages for different long bones b) Only occurs in bones of the skull c) Is permanent weak spot in bones d) Is technically a symphysis

A

Geniohyoid acts to: a) Pull the hyoid bone superiorly and has a vertical fiber direction b) Pull the hyoid bone inferiorly and has a vertical fiber direction c) Pull the hyoid bone superiorly and has a horizontal fiber direction d) Pull the hyoid bone inferiorly and has a horizontal fiber direction

A

Identify the False statement below pertaining to the Pronator Teres: a) It is innervated by the Ulnar Nerve b) It inserts in part on the Humerus c) It inserts upon the Radius d) It pronates the forearm

A

If one were to sustain a shearing force along the jugular foramen what assessment could you expect to indicative of cranial nerve damage/deficit? a. Inability to shrug the shoulders unilateral and/or bilateral b. Inability to chew or grind food c. Inability of the eye to follow finger up/down d. Inability to frown or pucker

A

In a typical thoracic vertebrae, the opening for the passage of the spinal cord is called the: a) Vertebral foramen b) Foramen transversarium c) Intervertebral foramen d) Foramen nervosum

A

In what order can you identify the anterior leg muscles distally medial to lateral? a) Tibialis anterior, extensor hallucis longus, extensor digitorum longus b) Extensor hallucis longus, extensor digitorum longus, tibialis anterior c) Tibialis anterior, extensor digitorum longus, extensor hallucis longus d) I don't know

A

Inflammation of the patellar ligament at the tibial tuberosity is called? a) Osgood-Schlatter Disease b) Baker's cyst c) Osteoarthritis d) Meniscal tear

A

Leaving work one day, you observe Prof. Freund walking across a busy intersection with her three year old child, Alex. Suddenly a crazed driver with a Maine license plate careens into the intersection. Prof Freund quickly pulls Alex onto the safety of the sidewalk, by tugging sharply upwards on her hand, with her arm outstretched overhead. Alex immediately cries; you suspect a brachial plexus injury. If your suspicion turns out to be true, it would most likely involve: a) The inferior trunk b) The elbow flexors c) The nerve roots of C5 and C6 d) The anterior scalene muscle

A

Lightly touch skin of inferior portion of occipital region of skull (below inion): Use the following list to match dermatomal and myotomal patterns. a) C2 b) C3 c) C4 d) C5 e) C6

A

Lymphedema is comprised of ___ while edema is mostly ___. a) Protein rich fluid, water b) Water, protein rich fluid c) Glucose rich fluid, carbohydrate rich fluid d) Carbohydrate rich fluid, glucose rich fluid

A

Match the Facial Action with the muscle that is the prime mover: Grinning (corners of mouth pulled laterally) a. Risorius b. Procerus c. Nasalis d. Depressor anguli oris e. Zygomatic major and minor

A

One action of the sternocostal head of the pectoralis major acts is: a) Adduction of the glenohumeral joint of the shoulder b) Lateral rotation of the glenohumeral joint of the shoulder c) Abduction of the glenohumeral joint of the shoulder d) Elevation of the upper ribs to assist with inspiration

A

Professor Cope skies into a sharp tree branch completely transecting the subclavian artery just under her right clavicle. Blood flow to the vessels would be compromised from the laceration and distally in the following order: a) Axillary, brachial, radial, ulnar b) Brachial, axillary, radial, ulnar c) Brachiocephalic, axillary, brachial, radial, ulnar d) Brachiocephalic, brachial, axillary, radial, ulnar

A

RESIST THE MVMT: Forward head/neck flexion: Use the following list to match dermatomal and myotomal patterns. a) C2 b) C3 c) C4 d) C5 e) C6

A

Riley is watching an old (B&W) episode of the Jack Lalane show (a fitness show from the 60's). While performing a partial sit up, bringing his left elbow to his right knee he can feel that the muscles assisting with this move are: a) Rectus abdominis (RA), right internal oblique (IO), left external oblique (EO) b) RA, left IO, right EO c) Transversus abdominis (TA) d) Bilateral external oblique

A

Rory says"The Tricuspid valve of the heart": a) Is located between the Right Atrium and Right Ventricle b) Is located between the Right Ventricle and Pulmonary Artery c) Is located between the Left Ventricle and Aorta d) Is located between the Left Atrium and Left Ventricle e) Is located between the Left and Right Atria

A

The Circle of Willis provides an important means of collateral circulation. What is the one vessel that is not a DIRECT part of this 'rotary circulatory system' of the brain? a. External carotid b. Posterior communicating c. Posterior cerebral d. Anterior communicating e. Basilar

A

The Pes Anserinus is formed (in part) from the tendon of: a. Sartorius b. Pectineus c. Rectus Femoris d. Vastus Medialis e. Adductor Magnus

A

The Superficial muscle group of the anterior aspect of the forearm includes: a) Pronator Teres b) Anconeus c) Flexor Pollicis Longus d) Flexor Digitorum Profundus e) Extensor Digitorum

A

The abdominal musculature (anterior and posterior walls) can be simultaneously contracted to assist in reducing stress/stabilizing the lumbar vertebrae a) True b) False

A

The abnormal condition of a winged scapula is associated with paralysis of the: a) Serratus anterior b) Rhomboids c) Latissimus dorsi d) Serratus posterior superior

A

The anterior wall of the axilla is formed by the: a) Pectoralis major b) Teres major and latissimus dorsi c) Serratus anterior d) Subscapularis and teres major

A

The axial skeleton is comprised of the: a) Skull, vertebral column, sternum and ribs b) Skull and vertebral column c) Skull, vertebral column and pelvis d) Scapula and bones of the limbs

A

The basilar artery feeds the ____________ artery at the base of the brain: a. Posterior cerebral b. Posterior communicating c. Middle cerebral d. Carotid

A

The brachial plexus injury known as Erb's palsy: a) Produces weakness in GH abduction b) Is also referred to as a lower trunk injury c) Involves the nerve roots of C6 and C7 d) Is often a result of catching oneself when falling from a height

A

The diaphragm is innervated by? a. Phrenic nerve b. Diaphragmatic nerve c. Vagus Nerve d. Spinal accessory nerve

A

The distal end of the radius articulates with: a) The scaphoid and the lunate b) The triquetrum and the pisiform c) The trapezoid and the trapezium d) The lunate and the cuboid

A

The distinguishing characteristic of a true rib is its: a) Connection to the sternum by its own costal cartilage b) Articulation with two adjacent vertebral bodies c) Articulation with the transverse process d) Absence of a neck or tubercle

A

The epidermis contains no blood vessels or lymph, while the dermis is richly vascularized and contains lymphatic vessels: a) True b) False

A

The following structure does NOT pass through the carpal tunnel : a) Flexor pollicis brevis tendon b) Lateral two tendons of flexor digitorum profundus c) Flexor pollicis longus tendon d) Median nerve

A

The head of a typical rib: a) Forms the costovertebral joint with vertebrae posteriorly b) Forms the costalchondral joint the sternum c) Forms the costotransverse joint with vertebrae posteriorly d) Attaches directly to the shaft of the rib, with no neck in between

A

The hepatic portal triad is comprised of: a. Bile duct, hepatic artery, portal vein b. Pancreatic duct, hepatic artery, splenic vein c. Bile duct, splenic artery, portal vein d. Pancreatic duct, splenic artery, splenic vein

A

The humeroulnar joint is an example of a: a) Uniaxial hinge joint b) Plane synovial joint c) Syndesmosis d) Ball and socket synovial joint

A

The infrahyoid muscles are in general innervated by: a) Branches of the ansa cervicalis b) Facial nerve c) Cranial nerves d) Terminal branches arising from the posterior cord of the brachial plexus

A

The internal carotid artery enters the basocranium and then transmits blood to the_____artery. a. Middle Cerebral b. Posterior Cerebral c. Anterior communicating d. Posterior communicating

A

The juncture between the esophagus and the stomach is called the: a) Z-line b) K-line c) H-line d) S-line

A

The large(st) thin(nest) muscle of facial expression that draws the inferior corners of the mouth downward is the: a. Platysma b. Masseter c. Buccinator d. Frontalis

A

The meningeal fold that divides the cerebral hemispheres into right and left is the: a. Falx Cerebri b. Falx Cerebelli c. Cerebellar Tentorium d. Crista Gallia

A

The muscles of the anterior abdominal wall do not assist with: a) Digestion b) Defecation c) Sneezing d) Partrution

A

The muscles that adduct the wrist include the: a) Flexor carpi ulnaris and extensor carpi ulnaris b) Flexor carpi radialis and extensor carpi radialis c) Extensor carpi radialis longus, extensor carpi radialis brevis, and flexor carpi radialis d) Brachioradialis, pronator teres, and pronator quadratus

A

The orbicularis oris muscle: a. Acts as a sphincter of the mouth b. Acts as a sphincter of the eye c. Allows one to smile d. Allows one to squint

A

The order of the 3 blood vessels arising from the aortic arch right to left is: a) Brachiocephalic, LCC, L Subclavian b) LCC, L Subclavian, & Vertebral off of subclavian c) Brachiocephalic, LSubclavian, LCC d) LCC, L Subclavian

A

The patella: a. Is the largest sesamoid bone in the human body b. Provides significant protection and stability to the knee joint anteriorly c. When removed the mechanical advantage of the quadriceps muscles is increased across the knee joint d. More than one of the above statements is accurate as related to the Patella

A

The phrenic nerve: a) Provides the sole motor innervation to the diaphragm b) Is derived solely from the rami of the brachial plexus c) Contains sympathetic fibers derived from the inferior cervical ganglion d) Enters the chest by passing anterior to the clavicle e) Runs posterior to the posterior scalene muscle

A

The portion of the quadriceps femoris muscle that does not have a proximal attachment on the femur is the: a. Rectus femoris b. Vastus lateralis c. Vastus intermedius d. Vastus medialis

A

The portions of the triceps brachii muscle that cross only one joint are the: a) Lateral and medial heads b) Short and long heads c) Long and lateral heads d) Short and medial heads

A

The primary motions or actions of the pectineus muscle include hip: a. Adduction and flexion b. Adduction and extension c. Abduction and flexion d. Abduction and extension

A

The primary muscle(s) that stabilizes or holds the pelvis level to allow the right foot to clear the ground during walking is the: a. Left gluteus medius and minimus b. Right gluteus maximus c. Left gluteus maximus d. Right gluteus medius and minimus

A

The rotator cuff muscles of the shoulder and the deeper gluteal muscles of the hip are similar in the following way: a. They help to stabilize the joints that they surround b. They all adduct and laterally rotate the extremity to which they are attached c. There is a labrum surrounding the margin of the joint to reduce friction d. Their distal attachments are all medial to the head and neck of the bone on which they insert

A

The statement is true about BOTH the teres minor and teres major muscles is: a) They both have attachments on the scapula b) They both pass deep to the long head of the triceps brachii muscle c) They both are innvervated by the axillary nerve d) They both assist with external rotation of the humerus at the glenohumeral joint

A

The stomach lies in which quadrant: a. Left upper b. Left lower c. Right upper d. Right lower

A

The trapezius can perform all of the following functions except: a) Protract the scapula b) Retract the scapula c) Elevate the scapula d) Depress the scapula e) Rotate the scapula

A

The ulnar nerve is at risk for compression as it crosses the elbow joint, where it lies between the: a) Olecranon and medial epicondyle b) Olecranon and lateral epicondyle c) Trochlea and capitulum d) Greater ulnar notch and the lateral epicondyle

A

The vertebral artery on the right side of the body is fed by which of the following arteries? a) Brachiocephalic b) Left common carotid c) Left Subclavian d) Arch of the aorta

A

The zygopophyseal (facet) joint between adjacent vertebrae is a: a) Plane synovial joint b) Syndesmosis joint c) Fibrocartilaginous joint d) Pivot synovial joint

A

To clinically test the Superior Rectus muscle, one would ask the patient to look a. Superiorly b. Inferiorly c. Laterally d. Medially

A

Trapezius muscle cannot perform one of the motions listed below. It is: a) Protraction of the scapula b) Retraction of the scapula c) Elevation of the scapula d) Depression of the scapula

A

Triceps coxae includes: a. Superior gemelli, inferior gemelli, obturator internus b. Piriformis, superior gemelli, inferior gemelli c. Obturator internus, obturator externus, quadratus femoris d. Superior gemelli, inferior gemelli, quadratus femoris

A

True False: A Jones fracture occurs at the base of the fifth metatarsal a) True b) False

A

True False: ANALOGY: Carpals is to hand as Tarsals is to foot. a) True b) False

A

True False: ANALOGY: Humerus is to upper extremity as Femur is to lower extremity. a) True b) False

A

True False: ANALOGY: Pectinate muscles is to Atria as Trabelculae Carnae is to Ventricle. a) True b) False

A

True False: Another name for a high ankle sprain is a syndesmosis spran a) True b) False

A

True False: As related to Fetal Circulation. A mixture of oxygenated and deoxygenated blood circulates throughout the body of the fetus a) True b) False

A

True False: As related to Fetal Circulation. Nan Tip: The foramen ovali typically closes at birth leaving behind a "thumb print" in the right and left atria called the fossa ovali. a) True b) False

A

True False: Auricles ("Cope's mud rooms") are pouch like appendages attached to the atria that are embryonic heart tube derivatives. a) True b) False

A

True False: Coxa valga is an increased angle of inclination, caused by decreased weight bearing in children, resulting in a longer leg and decreased efficiency of hip abductors a) True b) False

A

True False: Deltoid ligament is made up of the following components - anterir tibiotalar, tibionavicular, and tibiocalcaneal a) True b) False

A

True False: Fabella syndrome occurs when a small sesamoid bone forms in the posterior aspect of the knee within the tendon of the gastroc a) True b) False

A

True False: Genu valgum = knock knees, genu varum = bow legged a) True b) False

A

True False: Glut med and min perform abduction and medial rotation of the thigh a) True b) False

A

True False: Gravity and peristalsis helps move food through the esophagus a) True b) False

A

True False: Hammer toe is a hyperextended MTP and flexed PIP, caused by a volar plate injury a) True b) False

A

True False: Heartburn, fatigue on exertion and back pain are common symptoms related to coronary heart disease (CHD) in women. a) True b) False

A

True False: If pressure in compartment syndrome is not released, it can lead to necrosis. a) True b) False

A

True False: Legg-Calve-Perthes is avascular necrosis of the femoral head, resulting in decreased hip flexion, abduction, and internal rotation a) True b) False

A

True False: Morton's neuroma is plantar nerve compression between the metatarsals a) True b) False

A

True False: Peroneus longus and brevis evert the ankle and are responsible for counteracting medial sway during SLS a) True b) False

A

True False: Quadratus lumborum, external oblique, internal oblique, and iliacus have a common attachment on the iliac crest a) True b) False

A

True False: Retroversion results in toe-out to align the hips a) True b) False

A

True False: Right, left, quadrate, and caudate are lobes of the largest organ/gland in the human body a) True b) False

A

True False: SCFE (slipped capital femoral epiphysis) is fracture of the growth plate, caused by shearing with abduction and external rotation a) True b) False

A

True False: Sciatic nerve exits through the greater sciatic foramen and innervates no muscles in the gluteal region a) True b) False

A

True False: Tarsal bones are talus, calcaneus, navicular, cuneiforms I, II, III, and cuboid a) True b) False

A

True False: Testing for the status of the abducens nerve, the therapist asks the patient to follow their finger with their eyes as they draw their finger laterally across the patient's field of vision a) True b) False

A

True False: The L4-S2 branches of the lumbosacral plexus make up the fibular nerve a) True b) False

A

True False: The Levator ani group of the pelvic floor muscles include iliococygeus, pubococcygeus, & puborectalis a) True b) False

A

True False: The abdominal musculature (anterior, and posterior abdominal wall) can be simultaneously contracted to assist in reducing stress / stabilizing the lumbar vertebrae. a) True b) False

A

True False: The angle of Louis or sternal angle is an important landmark as it marks the location of the left main bronchus and the arch of the aorta ( and it is good to be able to speak with colleagues who perform cardiothoracic surgery). a) True b) False

A

True False: The angle of inclination is taken between the femoral shaft and neck in a frontal plane a) True b) False

A

True False: The anterior longitudinal ligament resists trunk extension a) True b) False

A

True False: The auricle is an appendage on both the left and right sides of the heart that allows for increased volume a) True b) False

A

True False: The brain is supplied by two main sources of blood, the vertebral artery and internal carotid artery a) True b) False

A

True False: The carotid sheath contains the common carotid artery, internal jugular vein, and the vagus nerve a) True b) False

A

True False: The conjoint tendon consists of the aponeurotic fibers of the transverse abdominus and the internal oblique a) True b) False

A

True False: The crista terminalis is a ridge occurring between the smooth and rough muscular wall of the right atrium. a) True b) False

A

True False: The distal attachment of subscapularis can be seen on the anterior part of the proximal humerus a) True b) False

A

True False: The external rotators of the hip at least include piriformis, superior, inferior gemellus, obturator externus and internus a) True b) False

A

True False: The femur and patella make up a plane joint a) True b) False

A

True False: The following muscles are innervated by the occulomotor nerve - superior rectus, medial rectus, inferior rectus, inferior oblique, levator palpabrae superioris a) True b) False

A

True False: The geniohyoid is innervated by the 12th CN a) True b) False

A

True False: The iliofemoral ligament is the strongest ligament in the body a) True b) False

A

True False: The inner chamber of the left ventricle of the heart is comprised of the trabeculae carnae a) True b) False

A

True False: The midfoot is made up of the following components - navicular, cuboid, cuneiforms I, II, III a) True b) False

A

True False: The mortise and tenon joint is made up of the distal tibia, distal fibula, and proximal talus a) True b) False

A

True False: The muscles that contribute to the lateral longitudinal arch of the foot include flexor digitorum brevis and abductor digiti minimi a) True b) False

A

True False: The muscles that contribute to the medial longitudinal arch of the foot include tibialis anterior, plantaris longus, abductor hallucis, and flexor ditigorum brevis a) True b) False

A

True False: The muscles that contribute to the transverse arch of the foot include plantaris longus and tibialis posterior a) True b) False

A

True False: The nerves that supply the hip joint include the femoral nerve, obturator nerve, superior gluteal nerve, and nerve to quadratus femoris a) True b) False

A

True False: The orbit is comprised of: zygomatic, lacrimal, ethmoid, frontal, nasal, maxilla, palatine, sphenoid a) True b) False

A

True False: The popliteal artery and vein, tibial nerve, and common peroneal nerve make up the popliteal fossa a) True b) False

A

True False: The posterior compartment of the leg is the largest compartment a) True b) False

A

True False: The rearfoot is made up of the following components - talus and calcaneus a) True b) False

A

True False: The sciatic nerve divides at the superior borders of the popliteal fossa at about mid thigh to be observed as the tibial and common fibular (peroneal) nerves a) True b) False

A

True False: The tail of the pancreas lies in the hilum region of the spleen a) True b) False

A

True False: The three parts of the pharynx are nasopharynx, oropharynx, and laryngopharynx a) True b) False

A

True False: The triceps coxae consists of the superior gemellus, inferior gemellus, and obturator internus a) True b) False

A

True False: There are five purely motor CNs - occulomotor, trochlear, abducens, spinal accessory, and hypoglossal a) True b) False

A

True False: There are four muscular layers in the sole of the foot a) True b) False

A

True False: Tibial division innervates long head and fibular division of the sciatic nerve innervates the short head of the Biceps Femoris muscle a) True b) False

A

True False: Turf toe is a sprain of the first metatarsal, caused by hyperextension a) True b) False

A

True False: Two salivary glands are submandibular and parotid a) True b) False

A

True False: vara = decreased angle, valga = increased angle a) True b) False

A

What if I lose my spleen? Which organ picks up the slack? a. Liver b. Kidneys c. Pancreas d. None of the above is correct, no organ can pick up the slack

A

What is the Pia Mater of the cranial meninges? a. Internal vasculated layer b. Thin intermediate layer c. Dura that makes up the infoldings d. None of the above

A

What is the function of the pancreas? a. Produces Insulin b. Stores Bile c. Digest chime d. Releases histamines

A

What is the name of a zone 2 injury to the fifth metatarsal? a) Jones fracture b) Avulsion c) Morton's toe d) Hammer toe

A

What is the term for a deltoid tear causing avulsion of the medial malleolus, shifting the talus laterally, then fracturing the lateral malleolus? a) Pott's fracture b) Tendinitis c) Baker's cyst d) Osteoarthritis

A

What makes the sartorius muscle unique in the body is that it: a. Is the longest muscle in the body b. Is the strongest muscle in the body c. Contributes to both hip extension and knee flexion d. Crosses two joints

A

What structure may be observed where a tendon passes over a bone and facilitates movement by reducing friction? a) Bursa b) Synovial sheath c) Aponeurosis d) Fascia e) Ligament

A

Which facet of the patellofemoral joint is most frequently affected by chondromalacia? a) Odd facet b) Superior lateral facet c) Inferior lateral facet d) Middle medial facet

A

Which muscle does the medial meniscus attach to posteriorly? a) Semimembranosus b) Biceps femoris c) Semitendinosus d) None of the bove

A

Which muscle inserts on the dorsal aspect of the base of the distal phalanx of the great toe? a) Extensor hallucis longus b) Extensor hallucis brevis c) Flexor hallucis longus d) Extensor digitorum

A

Which muscle is responsible for 'unlocking' the knee from full extension by laterally rotating the femur on the tibial plateau? a) Popliteus b) Gastroc c) Soleus d) Plantaris

A

Which of the following is correct pertaining to the sacral plexus? a. It is formed at least in part by the spinal nerves L4-S4. b. It is related to the posterior surface of the piriformis muscle. c. It is joined by the coccyx trunk. d. It cannot be compressed during parturition (child birth), resulting in lower limb pain. e. Most of its branches exit the pelvis through the lesser sciatic foramen

A

Which of the following is incorrect pertaining to IV discs? a) They are well vascularized by blood vessels b) They differ in thickness in different parts of the vertebral column c) The nucleus pulposus is primarily composed of water d) The annulus fibrosus is composed of concentric lamellae of fibrocartilage

A

Which of the following muscles does not extend the thigh at the hip joint? a) Rectus femoris b) Semitendinosus c) Glut max d) Biceps femoris

A

While coaching intramural softball Professor Josh Robbins, DPT, demonstrates a slide headfirst into third base, jamming his middle finger. He is seen by Joe Sperlak, DPTS who is working as a hand therapist. He observes that Professor Robbins has a disruption of the terminal extensor tendon resulting in: a) Mallet deformity b) Stenosing tenosynovitis c) Boutonniere deformity d) Osteochondrosis

A

While dissecting in the lab one day Lexi Byrd, DPTS asks: 'If an injury destroyed the function of the axillary nerve, what muscle would be left with intact function that could assist with glenohumeral abduction?' Monica Cardona, DPTS answers her correctly by stating 'it is of course......': a) Supraspinatus b) Middle trapezius c) Middle deltoid d) Subscapularis

A

While playing lacrosse, Professor Cope cradles the ball, which involves rotation of the trunk and internal and external rotation of the upper extremities in synchrony. This movement of the trunk and forearm occurs in the: a) Horizontal plane b) Coronal plane c) Frontal plane d) Sagittal plane

A

With an injury to the thoracodorsal nerve, Dr. Stetts would likely have difficulty with: a) Raising the weight of her body with her arms as would occur in climbing a rope b) Raising her arm overhead as if to ask a question c) Completing a wall push up d) Completing sidebending with ipsilateral rotation of her head

A

With an injury to the thoracodorsal nerve, a patient would likely have difficult with: a) Extending and internally rotating their humerus to reach for their Elon ID located in the back pocket of their pants on the same side b) Flexing and abducting their humerus to raise their arm, indicating completion of the exam c) Protracting their scapula as in completing a wall push up d) Side bending and rotating their head as in releasing some stress during a tough exam

A

You examine a patient after an automobile accident that resulted in a posterior dislocation of the hip joint. Which nerve are you most concerned might have been injured by such an accident? a. Sciatic b. Femoral c. Obturator d. Lumbosacral trunk e. Pelvic splanchnic

A

The deep fibular nerve innervates all of the muscles in which compartment of the leg?

Anterior

A muscle that attaches on the iliotibial tract (or band) is the: a. Gluteus medius b. Gluteus maximus c. Gluteus minimus d. Sartorius e. More than one of the above muscles has attachment on the IT Band

B

A newborn sustained a facial nerve paralysis as a consequence of her delivery with forceps. The infant is likely to demonstrate: a) Auditory loss b) Weakness of the muscles of facial expression c) Weakness of the muscles of mastication d) Weakness of rotation of the head to the opposite side and side bending to the same side

B

A patient named Haley is seen in a hospital setting shortly after sustaining trauma from a free rock climbing accident. She is having trouble breathing and Karen B., DPTS, determines she has a problem with her diaphragm. What nerve has likely been injured? a) Medial pectoral b) Phrenic c) Long thoracic d) Spinal accessory (CN XI)

B

A patient you are evaluating in the clinic is unable to elevate their shoulder girdle. You recall that the myotomes involved in shoulder girdle elevation are:___, the peripheral nerve involved is: ___ and the muscle weakness is in the: ___. a) C4, axillary, levator scapulae b) C4, spinal accessory, upper trapezius c) C3, spinal accessory, levator scapulae d) C3, axillary, upper trapezius

B

A source of blood supply to the head of the femur is from the: a. Medial and lateral gluteal arteries b. Medial circumflex artery c. Superior and inferior adductor arteries d. Medial and lateral collateral arteries

B

After a kyphosis screening for the spine, Micah recommends to a patient that they "squeeze a can between their shoulder blades". He tells the patient that the primary muscles involved in retraction of the scapula are the: a) Serratus anterior and the middle trapezius b) Middle trapezius and rhomboid major and minor c) Serratus anterior and serratus posterior superior d) Rhomboid major and minor and lattissimus dorsi

B

All of the following are infrahyoid muscles EXCEPT: a) Sternothyroid b) Mylohyoid c) Omohyoid d) Thyrohyoid

B

All of the following muscles have an attachment on the clavicle EXCEPT: a) Subclavius b) Middle scalene c) Trapezius d) SCM

B

All of the following statements about the erector spinae muscles are true EXCEPT: a) They are easily palpable in the lumbar region on most people b) They are the primary flexors of the vertebral column c) They are supplied by the posterior primary rami of the spinal nerves d) Their long muscle bundles run vertically, parallel to the spinous processes of the vertebrae

B

An injury to the ___ trunk of the brachial plexus which causes weakness or paralysis of the lateral rotators of the humerus is called ___; a) Upper/Klumpke's paralysis b) Upper/Erb's palsy c) Lower/Klumpke's paralysis d) Lower/Erb's palsy

B

Anterior displacement of the vertebral column following a break at the pars interarticularis is called: a) Spondylolysis b) Spondylolisthesis c) Colle's fracture d) Spina Bifida e) Green Stick Fracture

B

Blood drains from the external jugular vein into the ___ as it returns deoxygenated blood to the right atrium a) Brachiocephalic - superior vena cava b) Brachiocephalic - subclavian c) Internal jugular - brachiocephalic - superior vena cava d) Brachiocephalic - internal jugular

B

Blood supply to the lower extremity from superior to inferior runs from descending aorta to femoral via: a. External iliac - common iliac - b. Common iliac - external iliac - c. Internal iliac - common iliac - d. Common iliac - internal iliac -

B

By locating the xiphoid process and then palpating inferolaterally along the borders of ribs 7-10, you can identify the: a) Linea semilunaris b) Costal margin c) Tendinous intersections d) Internal oblique muscle

B

Dr Folger plays tennis all week and develops 'tennis elbow'. Beck recommends the use of a tennis elbow strap just distal to the elbow joint and resting the overused extensor muscles at their distal attachments and appropriately provides support with a splint / brace to the: a) Humeroulnar joint b) Humeroradial joint c) Radiocarpal joint d) Proximal radioulnar joint

B

Dr. Bailey endures a significant injury to the proximal musculocutaneous nerve while playing in a Lacrosse tournament. Hannah Buckingham, DPTS, sees him for an evaluation and knows that he would continue to have normal functioning of the ____ and problems with sensation on the_________. a) Biceps brachii, lateral aspect of his forearm b) Brachioradialis, lateral aspect of his forearm c) Biceps brachii, medial aspect of his forearm d) Brachioradialis, medial aspect of his forearm

B

Following a radical mastecomtomy of a female's left breast she is unable to abduct and lift her arm overhead. What structure do you suspect was injured during surgery? a) Axillary nerve b) Long thoracic nerve c) Suprascapular nerve d) Thoracodorsal nerve

B

Functions of the spleen include: a. Digestion of fats b. Store platelets c. Create insulin d. Store bile

B

Identify the muscle below that is not considered a part of the deep flexor/pronator group: a) Pronator Quadratus b) Flexor Carpi Radialis c) Flexor Pollicis Longus d) Flexor Digitorum Profundus e) More than one of the above are correct

B

Identify the statement that is FALSE regarding the erector spinae muscle group: a) It is easily palpable in the lumbar region on must people b) It has a primary function of flexion of the vertebral column c) It is supplied by the posterior primary rami of the spinal nerves d) Its long muscle bundles run vertically, parallel to the spinous processes of the vertebrae

B

If you are palpating the spine of the scapula then you are at what vertebral level? a) C7 b) T3 c) T8 d) L3

B

In performing a laminectomy for a patient with a deranged disk syndrome the surgeon would need to cut through which of the following ligaments: a) Ligamentum nuchae b) Ligamentum flavum c) Anterior longitudinal ligament d) Posterior longitudinal ligament

B

Lateral neck flexion: Use the following list to match dermatomal and myotomal patterns. a) C2 b) C3 c) C4 d) C5 e) C6

B

Match the Facial Action with the muscle that is the prime mover: Something smells horrible a. Risorius b. Procerus c. Nasalis d. Depressor anguli oris e. Zygomatic major and minor

B

Most fibers of the external oblique muscle: a) Run transversely b) Run inferomedially from their superior attachment c) Pass deep to the linea alba d) Run inferolaterally from their superior attachment

B

Myotomal testing for C1 C2 involves having the patient: a) Shrug their shoulders b) Flex their neck forward c) Laterally flex their neck d) Take in a deep breath

B

Myotomal testing for C8 involves asking the patient to: a) Take a breath b) Extend their thumb c) Extend digit three d) Extended the wrist

B

Radiographic findings of loss of disc height, irregular end plates, separated epiphysis, and Schmorl's nodes in a person with a thoracic kyphosis are indicative of: a) Scoliosis b) Scheuermann's disease c) Excessive studying d) Spondylolisthesis

B

Samantha, DPTS sees a patient named Allison, who (FOOSH'd) fell on an outstretched hand and sustained a Colles fracture. She reviews the the radiograph (x-ray), and sees: a) Volar displacement of the distal fragment of the radius b) Dorsal displacement of the distal fragment of the radius c) Dislocation of the radial head d) Fracture of the scaphoid

B

Sarah Benson, DPTS, sees Dr. Chinworth in an outpatient clinic following a traumatic injury. She completes a manual muscle test, and documents that she had weakness in the triceps brachii and the wrist extensors. Based on this pattern of weakness, you suspect the injury is: a) To the lateral cord of the brachial plexus b) To the radial nerve c) To the spinal cord at the level of C8 d) Ro the musculocutaneous nerve

B

Sarah, DPT knows that flexing the distal phalanx of the little and ring finger tests the function of: a) Radial nerve b) Ulnar nerve c) Median nerve d) Musculocutaneous nerve

B

Scott, DPTS tells us "If my patient Claire avulsed her entire greater tubercle away from her humerus the following movements at the glenohumeral joint would be effected" a) Flexion and abduction b) Abduction and lateral rotation c) Flexion and medial rotation d) Extension and medial rotation e) Flexion, abduction, and adduction

B

Sensation to the tip of the index finger is provided by: a) Ulnar nerve b) Median nerve c) Radial nerve d) Musculocutaneous nerve

B

Stephen, tells us that the coronary sinus empties into the: a) Great Cardiac Vein b) Right Atrium c) LAD d) Left Atrium

B

The 2 anterior neck muscles that change direction via a pulley/tendon are: a) The digastric and sternohyoid muscles b) The digastric and omohyoid muscles c) The omohyoid and mylohyoid muscles d) The sternothyroid and digastric muscles

B

The Falx Cerebelli: a. Separates the Cerebral Hemispheres from one another b. Separates the Cerebellar Hemispheres from one another (at least in part) c. Separates the Cerebellum from the spinal cord d. Separates the Cerebrum from the Cerebellum

B

The Sternal Angle (AKA Angle of Louie) is located between the: a) Sternum and clavicle b) Manubrium and body of the sternum c) Body of the sternum and Xiphoid process d) Manubrium and Rib 1

B

The acetabular labrum a. Is fibrocartilaginous and helps prevent fractures of the 'true' femoral neck b. Is fibrocartilaginous and helps to deepen the socket for the femoral head c. Contains a small artery that can help supply blood to the head of the femur d. Is fibroadipose and helps the head of the femur to dislocate as needed for motion

B

The acromioclavicular joint is a: a) Saddle synovial joint b) Plane synovial joint c) Secondary cartilaginous joint d) Syndesmosis joint

B

The adductor canal is: a. An opening in the adductor magnus muscle. b. A mid-thigh intermuscular passageway that conducts the femoral vessels. c. The space between the adductor longus and brevis that transmits the anterior divisions of the obturator vessels. d. The passageway used by the obturator internus muscle to exit the pelvis. e. The passageway used by the obturator externus muscle to reach the greater trochanter

B

The adductor hiatus: a. Begins at the apex of the femoral triangle b. Transmits femoral artery and vein from anterior medial thigh to the popliteal fossa c. Is formed in part by an opening in gracilis d. More than one of the above are correct

B

The anatomical landmarks that are located from inferior to superior, respectively, include a) Conoid tubercle, coracoid process, coronoid fossa b) Coronoid fossa, coracoid process, conoid tubercle c) Coronoid fossa, conoid tubercle, coracoid process d) Coronation fossa, coracoid process, conoid tubercle

B

The appendix is located on what portion of the large intestine? a) Transverse b) Cecum c) Descending d) Sigmoid

B

The bone which forms the cheekbones and lateral portion of the orbit is the: a. Mandible b. Zygomatic c. Maxilla d. Frontal

B

The common fibular nerve splits around which landmark? a) Head of the tibia b) Head of the fibula c) Lateral condyle of the tibia d) Medial condyle of the tibia

B

The common inferior and medial attachment of the internal oblique and transversus abdominus muscles is the: a) Inguinal ligament b) Conjoint tendon c) Anterior border of ribs 5-12 d) Arcuate line

B

The disorder characterized by progressive fibrosis or disease of the palmar aponeurosis is: a) DeQuervain's disease b) Dupuytren's contracture c) Trigger finger d) Carpal tunnel syndrome

B

The erector spinae muscle group is made up of three columns. From lateral to medial they are: a) Splenius, longissimus, iliocostalis b) Iliocostalis, longissimus, spinals c) Iliocostalis, longissimus, splenius d) Spinalis, longissimus, iliocostalis

B

The femoral triangle is bordered on the lateral side by the: a. Inferior border of the inguinal ligament b. Medial border of the sartorius muscle c. Lateral border of the pectineus muscle d. Medial border of the adductor longus muscle

B

The femur and the tibia make what type of joint? a) Ball and socket b) Hinge c) Plane synovial d) Gliding

B

The fibers of the external intercostal muscles: a) Run mostly in the same direction as the transverse abdominis muscles b) Run mostly in the same direction as the external oblique muscles c) Attach to the outsides of ribs 5-12 d) Have a membranous attachment between the neck of the rib and the vertebrae

B

The following statements are true about BOTH the rhomboid minor and rhomboid major muscles EXCEPT: a) They both have attachments on the scapula b) They both lie inferior to the serratus posterior superior muscle c) They both have attachments on the spinous processes in the thoracic region d) They both assist with retraction of the scapulae

B

The hip ligament that is LEAST important in providing stability or support to the hip joint is the: a. Iliofemoral ligament b. Ligament to the head of the femur, (ligamentum teres) c. Pubofemoral ligament d. Ischiofemoral ligament

B

The humerus has a: a) Biceps muscle attachment b) More prominent medial epicondyle than lateral epicondyle c) Greater tubercle located medial to the lesser tubercle d) Coronoid fossa located on its posterior surface

B

The iliofemoral and pubofemoral ligaments have the following in common: a. Lateral attachment on the lesser trochanter b. They both limit hyperextension of the femur at the hip joint c. They both limit hyperflexion of the femur at the hip joint d. Lateral attachment on the intertrochanteric crest e. More than one of the above is correct

B

The intertubercular groove is: a) Also called the bicipital groove because of the tendon attaching IN IT b) Bordered by the lesser tubercle medially and the greater tubercle laterally c) It is located on the posterior portion of the humerus d) It is covered superficially by the pectoralis minor muscle

B

The joint capsule of the hip encompasses the following structures: a. Head and neck of the femur b. Head and neck of the femur and acetabular labrum c. Head, neck and greater and lesser trochanters of femur and acetabular labrum d. Head, neck and greater and lesser trochanters of femur

B

The main function of the infrahyoid muscle group is to: a) Form the floor of the oral cavity b) Depress the hyoid bone during swallowing and speaking c) Move the larynx superiorly during swallowing d) Stabilize the sternum

B

The most commonly damaged portion of the "rotator cuff" is the tendous portion of: a) Infraspinatus b) Supraspinatus c) Teres Minor d) Deltoid

B

The muscle that acts as a sphincter of the eye is the: a. Frontalis b. Orbicularis oculi c. Orbicularis oris d. Mentalis

B

The muscle that has primary actions of scapular protraction and upward rotation is the: a) Levator scapulae b) Serratus anterior c) Upper portion of the trapezius d) Rhomboid major

B

The muscle that has primary actions of scapular protraction and upward rotation of the glenoid fossa is the: a) Levator scapulae b) Serratus anterior c) Upper portion of the trapezius d) Rhomboid major

B

The origin of the anterior cerebral artery is ______________ and it provides blood supply to the ____________ region of the brain. a. Basilar artery / frontal lobes of the cerebral hemisphere b. Internal carotid artery / frontal lobes of the cerebral hemisphere c. Basilar artery / occipital lobe of cerebral hemisphere d. Internal carotid artery / occipital lobe of cerebral hemisphere

B

The parotid duct passes through which muscle of the face? a) Masseter b) Buccinator c) Orbicularis oris d) Levator palpebrae superioris

B

The patient you are working with had an injury to the nerve supplying the gluteus medius muscle. Under these circumstances, the patient is likely to also demonstrate weakness of the: a. Gluteus maximus muscle b. Tensor fascia latae muscle c. Obturator internus muscle d. Adductor brevis muscle

B

The pectoralis minor muscle acts to: a) Draw the clavicle medially b) Stabilize the scapula by drawing it inferiorly and anteriorly against the thoracic wall c) Protract the scapula and rotate it upward d) Adduct and medially rotate the humerus at the shoulder

B

The portion of the skull that houses the brain and cranial meninges is called the: a. Mandible b. Neurocranium c. Facial skeleton d. TMJ

B

The primary muscles involved in retraction of the scapula are the: a) Serratus anterior and the middle trapezius b) Middle trapezius and rhomboid major and minor c) Serratus anterior and serratus posterior superior d) Rhomboid major and minor and latissimus dorsi

B

The principle flexor of the forearm at the elbow: a) Inserts upon the radius b) Lies deep to Biceps c) Receives motor innervation from the Ulnar Nerve d) Originates from the scapula

B

The role of the ___ sphincter is to control movement of chyme from the stomach into the small intestine (duodenum) a) Cardiac b) Pyloric c) Urethral d) Anal

B

The rotator cuff muscle that contributes the least to med/lat rotation of the humerus at the glenohumeral joint is the: a) Subscapularis b) Supraspinatus c) Teres minor d) Infraspinatus

B

The rotatores: a) Have a relatively large mechanical advantage for producing trunk rotation b) Possibly function primarily as "kinesiological monitors" or organs of proprioception and assist in stabilization of the spine with multifidus c) Are part of the erector spinae group of muscles d) When acting bilaterally, can produce trunk flexion

B

The scalenus anterior muscle typically: a) Inserts on the second rib b) Runs anterior to the rami of the brachial plexus c) Is innervated by the phrenic nerve d) Descends posterior to the subclavian artery

B

The skin dimples overlying the PSIS lie at what vertebral level? a. S1 b. S2 c. S3 d. S4

B

The sternal angle of angle of Louis, is located between the: a) Body of the sternum and the xiphoid process b) Manubrium and the body of the sternum c) Sternum and the clavicle d) The left and right portions of the sternocleidomastoid muscle

B

The strongest flexor of the hip is the: a. Rectus femoris b. Iliopsoas c. Sartorius d. Biceps femoris

B

The suboccipital triangle musculature includes all of the following EXCEPT: a) Rectus capitus posterior major b) Rectus cervicis posterior minor c) Obliquus capitis superior d) Obliquus capitis inferior

B

The ventral ramus (root) that forms the middle trunk of the brachial plexus is: a) C6 b) C7 c) C8 d) T1

B

This pathology is characterized by pain in the kneecap when flexing the leg, sometimes referred to as Runner's Knee a) Osgood-Schlatter Disease b) Chondromalacia patella c) Baker's cyst d) Osteoarthritis

B

To stretch the quadriceps muscles, you would: a. Extend the hip and extend the knee b. Extend the hip and flex the knee c. Flex the hip and extend the knee d. Flex the hip and flex the knee

B

True False: 22. In general Obturator innervates the adductors and Femoral innervates the flexors of the leg at knee a) True b) False

B

True False: 23. In subadults, the angle of inclination ranges between 90-120 degrees a) True b) False

B

True False: ACL injury is tested by posterior drawer test a) True b) False

B

True False: ANALOGY: Bicuspid is to Left AV valve as Mitral Valve is to Right AV valve. a) True b) False

B

True False: ANALOGY: Digit 3 of hand is to midline in hand as Digit 3 of foot is to midline in foot. a) True b) False

B

True False: Below the arcuate line, the aponeurosis of the internal oblique and the transverse abdominus lie posteriorly to the rectus abdominus a) True b) False

B

True False: Blood supply to the spleen is from the hepatic artery a) True b) False

B

True False: Genu = hip, coxa = knee a) True b) False

B

True False: If you lost tibial innervation, the lateral compartment would be affected a) True b) False

B

True False: In a posterior dislocation, typically caused by a traumatic accident, the acetabulum lies posterior to the femoral head a) True b) False

B

True False: Intrinsic back muscles include serratus posterior inferior and superior a) True b) False

B

True False: Inversion and eversion occur at the forefoot a) True b) False

B

True False: LCL, MCL, ACL, and PCL are intracapsular and are nutritionally "fed" by the articular capsule a) True b) False

B

True False: Muscles of mastication include: temporalis, masseter, buccinator, medial pterygoids, lateral pterygoids, orbicularis oris a) True b) False

B

True False: Orbicularis oculi is a sphincter muscle that acts to pucker the lips around the mouth a) True b) False

B

True False: Pes planus = high arches, pes cavus = flat arches a) True b) False

B

True False: Piriformis syndrome is when the tibial nerve pierces the piriformis muscle instead of running inferiorly to it a) True b) False

B

True False: Psoas minor flexes the hip a) True b) False

B

True False: Rugae are folds that allow for expansion of the small intestine a) True b) False

B

True False: Sciatic nerve is comprised of L4-S3 and exits the pelvis through the lesser sciatic notch a) True b) False

B

True False: Shin splints are caused by overuse and repetitive concentration contractions of the tibialis anterior and/or tibialis posterior a) True b) False

B

True False: The ACL is the most important ligament in the knee, providing a central axis for rotation a) True b) False

B

True False: The AV node is the primary pace maker of the heart. a) True b) False

B

True False: The axillary nerve innervated teres major a) True b) False

B

True False: The floor of the inguinal canal is made up of aponeurosis of the internal oblique a) True b) False

B

True False: The fossa ovalis is a depression located in the interventricular septum and is the former site of the foramen ovale in the fetus. a) True b) False

B

True False: The gastroc and soleus plantarflex the ankle while the knee is flexed and raise the heel during walking a) True b) False

B

True False: The innervation of lumbrical I and II in the foot is medial plantar nerve a) True b) False

B

True False: The lateral meniscus is more "O"-shaped and moves more freely than the medial meniscus a) True b) False

B

True False: The lateral pectoralis nerve supplies both pectoralis major and minor a) True b) False

B

True False: The midfoot is made up of the following components - metatarsals and phalanges a) True b) False

B

True False: The number of tarsal bones in the foot is the same as that of the carpal bones in the wrist. a) True b) False

B

True False: The portion of the large intestine that demarcates the upper form the lower quadrants is the ascending colon a) True b) False

B

True False: The role of the pyloric sphincter is to prevent regurgitation from the stomach back up into the esophagus a) True b) False

B

True False: The skull is made up of the facial and viscerocranium a) True b) False

B

True False: The superficial layer of the forearm flexors includes pronator quadratus a) True b) False

B

True False: The superior rectus runs through a pulley called the trochlea that is attached to the frontal bone a) True b) False

B

True False: The third layer of the foot contains the lumbricals, quadratus plantae, and tendons of FHL and FDL a) True b) False

B

True False: There are four arches of the foot a) True b) False

B

True False: To test the glossopharyngeal nerve, the therapist asks the patient to stick out their tongue a) True b) False

B

True False: Trendelenburg is ipsilateral hip drop due to weak gluteus medius and minimus a) True b) False

B

True False: Upper and lower surascapular nerve innervates subscapularis a) True b) False

B

What foramen/aperture does Cranial Nerve VII exit? a. Foramen Spinosum b. Stylomastoid Foramen c. Foramen Rotundum d. Superior Ovale Fissure

B

What is the term for patient presentation of digit II in the foot being longer than digit I? a) Morton's neuroma b) Morton's toe c) Turf toe d) Jones fracture

B

When you "just say no" to telemarketers at the front door lobbying to speak to you, the primary motion occurring at the articulation between your atlas and axis is: a) Lateral flexion b) Rotation c) Flexion and extension d) Abduction

B

Where does the majority of the absorption of nutrient take place in the small intestines? a. Duodenum b. Jejunum c. Ceum d. Illeum

B

Which muscle plantarflexes and inverts the ankle? a) Tibialis anterior b) Tibialis posterior c) Gastroc d) Plantaris

B

Which muscles adducts the fingers toward midline? a) Lumbricals b) Palmar interossei c) Dorsal interossei d) Opponens pollicis

B

Which of the following is NOT a muscle of mastication? a. Medial Pterygoid b. Orbicularis Oris c. Lateral Pterygoid d. Temporalis e. Masseter

B

Which of the following is incorrect pertaining to the inguinal canal? a) It can transmit an indirect inguinal hernia b) Its role is to transmit structures from the abdominal to the thoracic cavity c) Its superficial opening is the superficial (external) inguinal ring that is the passage for spermatic cord in males d) Its floor is formed by the inguinal ligament

B

Which of the following muscles is NOT innervated (at least in part) by the Obturator Nerve? a. Adductor Longus b. Tensor Fascia Latae c. Adductor Magnus d. Gracilis e. Obturator Externus

B

Which of the following pairs of planes would pass through both arms simultaneously, with the person standing in normal anatomical position? a) Sagittal and frontal b) Transverse and frontal c) Transverse and sagittal d) Longitudinal and median

B

Which of the following statements is TRUE? a) In adults greater than 50 yo, rib fractures are the most common type of fracture b) Colle's fracture is a fracture of the distal radius c) The lunate is the most commonly fractured carpal bone d) Fractures of the humerus are referred to as "boxer's fractures"

B

Which of the following structures is not connected to the hyoid bone by a muscle? a) Styloid process of the temporal bone b) Inion, or occipital protuberance c) Scapula d) Mastoid process e) Mandible

B

Which portion of the quadriceps femoris muscle has a proximal attachment on the greater trochanter and lateral lip of the linea aspera of the femur? a. Rectus femoris b. Vastus lateralis c. Vastus medialis d. Vastus intermedius

B

Which statements are true about the dermis layer? It contains: a) Stored fat cells b) Hair follicles and arrector muscles that give you goose bumps c) Skin ligaments d) All of the above

B

While water skiing on the Haw River, Dr. Freund injured her medial pectoral nerve. Hannah sees Dr. Freund in the skills lab and believe that this injury would likely result in: a) Complete paralysis of pectoralis major b) Complete paralysis of pectoralis minor c) Partial paralysis of subclavius d) Partial paralysis of serratus anterior

B

You have just evaluated a patient and determined that she has a torticollis creating a chronic muscle spasm of the left SCM muscle. The position the patient is likely to hold her head in is: a) Rotation of the head to the left and lateral flexion to the left b) Rotation of the head to the right and lateral flexion to the left c) Rotation of the head to the right and lateral flexion to the right d) Rotation of the head to the left and lateral flexion to the right

B

You see a patient who has tightness and inflammation of the piriformis muscle. The nerve bundle that is likely to be affected by this inflammation is the: a. Obturator b. Sciatic c. Femoral d. Superior gluteal

B

Your patient is status post lumbar fusion surgery. You arrive to her room for Day 1 acute physical therapy session. The physical therapy referral states "Evaluate and Treat, Bed Mobility & Ambulate". The patient is sitting in the hospital bed with the lights off complaining of a severe headache. She is pleading that she have something for pain prior to therapy. What is the cause of headache and what would be your course of action? PICK B. The patient could possibly have a CSF leak from the surgical procedure causing the headache; Contact the physician and hold therapy until patient is appropriate to resume. a. The patient is drug seeking and hates physical therapy; Document in their chart "Attempted evaluation, not appropriate for therapy" b. The patient could possibly have a CSF leak from the surgical procedure causing the headache; Contact the physician and hold therapy until patient is appropriate to resume. c. The patient could possibly have a CSF leak from the surgical procedure causing the headache; evaluate the patient and document your findings

B

Zac, DPT and neurosurgeon, knows that to gain access to the spinal cord for a surgical procedure, he will most likely have to perform a? a) Electromyelogram b) Laminectomy c) Retinacular Release d) MRI

B

A dermatome and myotome represent the following in relationship to the spinal cord: segmental a) Cutaneous innervation of skin and peripheral nerve testing b) Peripheral muscle innervations and periphereal sensory innervations c) Cutaneous innervations of skin and specific muscle (motor) activities that represent a spinal segment d) Peripheral nerve innervations to skin and muscles

C

A distinguishing feature of the atlas (C1) NOT present on other cervical vertebrae is the presence of the: a) Dens b) Foramen transversarium c) Facet for dens d) Vertebral body

C

A fracture to the midshaft of the humerus creates a high risk of injury to the: a) Musculocutaneous nerve b) Midhumeral circumflex nerve c) Radial nerve d) Lateral nerve

C

A patient is seen in a rehabilitation clinic with winging of the scapula, and weakness in upward rotation and protection of the scapula. The nerve that is likely to have been injured is the: a) Suprascapular nerve b) Infrascapular nerve c) Long thoracic nerve d) Short thoracic nerve

C

A patient lying face up on a table is: a) In normal anatomical position b) Lying prone c) Lying supine d) Sidelying

C

A patient with a chronic lung disorder and labored breathing might be able to assist with elevation of the second rib during inspiration through contraction of the: a) Levator scapula b) Subclavius medius c) Scalenus posterior d) Semispinalis cervicis

C

A role of the abdominal muscles is: a) Extension of the spine b) Primary muscles of respiration c) Support and protect abdominal viscera d) Flexion of the head

C

A weight-bearing surface in sitting is the: a. Gluteus maximus b. Ischial ramus c. Ischial tuberosity d. Ischial spine

C

Abduction of the shoulder (glenohumeral joint) means movement of the arm: a) Away from the midline of the body, in the median plane b) Toward the midline of the body, in the frontal/coronal plane c) Away from the midline of the body, in the frontal/coronal plane d) Toward the midline of the body, in the median plane

C

All of the following statements about the intertubercular groove of the humerus are true EXCEPT: a) It is also called the bicipital groove because of the tendon passing through it b) The latissimus dorsi attaches to the floor c) It is located on the posterior portion of the humerus d) It is covered superficially by the pectoralis major muscle

C

All of these muscles can act to contribute to rotation of the spine/trunk EXCEPT: a) Internal oblique b) Rotatores c) Transverse abdominus d) External oblique

C

An injury to the ___ trunk of the brachial plexus which causes weakness or paralysis of the wrist and fingers is called ___; a) Upper/Klumpke's paralysis b) Upper/Erb's palsy c) Lower/Klumpke's paralysis d) Lower/Erb's palsy

C

Below the arcuate line the rectus abdominis is covered anteriorly by the: a) Aponeurosis of EO, ½ of IO and TA b) Aponeurosis of ½ of IO and TA c) Aponeurosis of EO, IO, TA d) Parietal peritoneum

C

Blood supply to the face is by: a. Internal carotid artery b. Frontal artery c. External carotid artery d. Nasalis artery

C

Dermatome S4 is located at the a. Adductor region b. Medial thigh c. Perianal region d. Big toe

C

Dr. Cope is carrying a scalpel in her back right pocket when she leans against the counter in the anatomy lab. Dr. Zimmerman suspects that the following muscles are weak or not working? a. Rectus femoris and pectineus on the right b. Rectus femoris and pectineus on the left c. Gluteus medius and minimus on the right d. Gluteus medius and minimus on the left

C

Dr. Zimmerman is seeing a patient who has had recent shoulder surgery to stabilize the head of the humerus at the glenohumeral joint. The rotator cuff muscles that help to stabilize the head of the humerus at the GHJ include: a) Deltoid, supraspinatus, infraspinatus, teres minor b) Deltoid, supraspinatus, infraspinatus, teres major c) Supraspinatus, infraspinatus, teres minor, subscapularis d) Supraspinatus, infraspinatus, teres major, subscapularis

C

During exhalation: a) The diaphragm contracts and the ribs move laterally up and out b) The diaphragm relaxes and the ribs move laterally up and out c) The diaphragm relaxes and the ribs move medially down and in d) The diaphragm contracts and the ribs move medially down and in

C

Early in the post-operative management of a patient with a total hip replacement (posteriolateral surgical approach), the patient should be taught to avoid the movement of: a. Hip extension beyond 90 degrees because of increased chance of dislocation b. Hip abduction beyond 30 degrees because of increased chance of avascular necrosis c. Hip flexion beyond 90 degrees because of increased chance of dislocation d. Hip adduction beyond 30 degrees because of increased chance of avascular necrosis

C

Elevation of the scapula is performed primarily by the: a) Upper trapezius and splenius capitus b) Levator scapulae and splenius capitus c) Upper trapezius and levator scapulae d) Pectoralis minor and upper trapezius

C

Emily Fonke, DPTS, knows that early treatment for an office worker with symptoms of carpal tunnel syndrome would NOT include: a) Adjustment of office furniture to maintain the wrists in a more neutral position while typing b) Change in his work-rest schedule to allow a regular pause away from the keyboard c) High force, high repetition strengthening exercises to counter the effects of disuse d) Night splinting to keep the wrists in a neutral position during sleep

C

Following an ice pick wound to the neck, the most significant physical issue that you observe is that the patient's medial border of his left scapula is located further from the midline than that of his right scapula. He is also weak in scapula elevation. The patient is able to depress his scapula, medially rotate & extend his humerus, and abduct his humerus. He is also able to rotate his head to the right and laterally flex his head to the left shoulder. The nerve most likely injured by the wound is the: a) Thoracodorsal b) Suprascapular c) Dorsal scapular d) Spinal accessory

C

How would you test the sensory component of the glossopharyneal nerve? a. Snell Eye chart b. Assessing the patient ability to distinguish the smell of peppermint c. Assessing taste of the posterior 1/3 of the tongue d. Assessing the taste of the anterior 2/3 of the tongue

C

IT tract or band has distal attachment at: a. Pes anserine b. Medial tibial plateau c. Gerdy's tubercle / anterolateral tibial condyle d. Head of fibula

C

Identify the False statement below regarding the median nerve: a) It is derived from the lateral and medial cords of the Brachial Plexus b) It innervates muscles in the forearm and hand c) It provides cutaneous innervation to the MOST medial digit dorsally d) It innervates the second Lumbrical e) It may be involved in Carpal Tunnel Syndrome

C

In what order can you identify the posterior leg muscles distally starting at the medial malleolus and moving medial to lateral? a) Tibialis posterior, tibial nerve and artery, flexor hallucis longus, flexor digitorum longus b) Tibial nerve and artery, tibialis posterior, flexor digitorum longus, flexor hallucis longus c) Tibialis posterior, flexor digitorum longus, tibial nerve and artery, flexor hallucis longus d) I don't know

C

In which direction does the patella typically displace? a) Supermedially b) Inferolaterally c) Superolaterally d) Inferomedially

C

Inflammation of synovial folds, most often medially is called? a) Osgood-Schlatter Disease b) Baker's cyst c) Plica Syndrome d) Osteoarthritis

C

Jeremy, DPTS sees a patient in a rehabilitation clinic with winging of the scapula, and weakness in upward rotation and protraction of the scapula. The nerve that is likely to have been injured is the: a) Suprascapular nerve b) Infrascapular nerve c) Long thoracic nerve d) Short thoracic nerve

C

Lightly touch at and above clavicular region: Use the following list to match dermatomal and myotomal patterns. a) C2 b) C3 c) C4 d) C5 e) C6

C

Linea semilunaris is the name Arnold tells us describes the: a) Curved line demarcating the change in the composition of the rectus sheath b) Three horizontal lines crossing the superior half of the rectus abdominis c) Lateral borders of the rectus abdominis d) Indentation formed by the inferior border of the rib cage

C

Match the Facial Action with the muscle that is the prime mover Flare nostrils as in coming up for air whilst swimming the butterfly stroke a. Risorius b. Procerus c. Nasalis d. Depressor anguli oris e. Zygomatic major and minor

C

Myotomal testing for C7 involves having the patient: a) Take a breath b) Perform shoulder abduction c) Extended the elbow and flex the wrist d) Flex the elbow and extend the wrist

C

Radiate ligaments perform each of the following except: a) Stabilizes the head of the rib posteriorly b) Stabilizes the ribs anteriorly at the sternocostal articulation c) Stabilizes the odontoid process between C1 and C2 d) Stabilizes the joint capsule of the head of ribs 1-12

C

Retired Dean Rogers is an avid cyclist, who typically averages 100 miles/day on the weekends, and 30-40 miles/day during the week. She recently increased her miles in training for a cross-country tour. She has been referred to you with a compression syndrome of the ulnar nerve at Guyon's canal. The muscle that will continue to be working because it is not compromised is the: a) Abductor digiti minimi b) Adductor pollicis c) Flexor carpi ulnaris d) Palmar interossi

C

Scapulohumeral rhythm refers to: a) The changing articluar configuration of the humeral head in the glenoid cavity with movement b) The protective contractions of the rotator cuff muscles with humeral abduction c) The relationship between movements of the glenohumeral and scapulothoracic joints during humeral abduction d) The association between the movements at the glenohumeral and acromioclavicular joints

C

Shrug shoulders towards ceiling: Use the following list to match dermatomal and myotomal patterns. a) C2 b) C3 c) C4 d C5 e) C6

C

The "M" of the Brachial Plexus is formed (in part) from which of the following nerves? a) Radial b) Axillary c) Ulnar d) Long Thoracic

C

The Median Nerve supplies motor innervation to ALL of the following muscles EXCEPT: a) Abductor Pollicis Brevis b) Flexor Pollicis Longus c) Adductor Pollicis d) Lumbrical #2 e) Opponens Pollicis

C

The Ober test is used to examine: a. Hip joint integrity b. Pubic symphysis integrity c. Iliotibial tract tightness d. Hip flexor tightness

C

The Oculomotor Nerve does NOT provide motor innervation to: a. Levator Palpebrae b. Inferior Oblique c. Orbicularis Oculi d. Medial Rectus

C

The attachments of the deltoid include deltoid tuberosity and: a) Clavicle b) Clavicle, spine of scapula c) Clavicle, spine of scapula, acromium d) Clavicle, spine of scapula, acromium, sternum

C

The brachiocephalic veins a) Receive blood directly from the right subclavian artery b) Receive blood directly from the right and left facial veins c) Empty into the superior vena cava d) Empty into the inferior vena cava

C

The coracoid process of the scapula: a) Projects posteriorly and slightly laterally from the superior border of the scapula b) Articulates (forms a joint) with the clavicle c) Is the superior attachment of the pectoralis minor muscle d) Is the superior attachment of the pectoralis major muscle

C

The crista galli is a thick triangular process projecting superiorly from the __________ bone and anchoring the _____________portion of the meningeal folds a. Zygomatic / falx cerebri b. Zygomatic/ falx cerebelli c. Ethmoid / falx cerebri d. Ethmoid / falx cerebelli

C

The diaphragm is innervated by the ___ nerve which originates from spinal nerves ___ a) Phrenic, C2-6 b) Inferior ansa cervicalis, C2-3 c) Phrenic, C3-5 d) Lesser occipital, C2

C

The distal attachment of the psoas major muscle is: a. The tendon of the psoas minor muscle to the body of the femur b. The greater trochanter of the femur c. The lesser trochanter of the femur d. The pectineal line, iliopectineal eminence via iliopectineal arch on the pubic bone

C

The following two actions are common to the biceps femoris, semitendinosus, and the semimembranosus: a. Hip extension and knee medial rotation b. Hip flexion and knee flexion c. Hip extension and knee flexion d. Hip flexion and knee extension

C

The head of the fibula articulates with the: a. Dome of the talus b. Medial tibial condyle c. Lateral tibial condyle d. Medial epicondyle of the femur

C

The head of the radius articulates with: a) The ulnar head b) The coracoid process c) The capitulum d) The olecranon

C

The hypothenar muscles include the: a) Abductor pollicis brevis, flexor pollicis brevis, and opponens pollicis b) Abductor digiti minimi, and flexor digiti minimi (brevis) c) Abductor digiti minimi, flexor digiti minimi (brevis), and opponens digiti minimi d) Abductor digiti minimi, flexor digiti minimi (brevis), opponens digiti minimi, and adductor digiti minimi

C

The iliotibial tract is the conjoint distal aponeurotic attachment of which of the following pairs of muscles? a. Gluteus medius and minimus b. Gluteus medius and maximus c. Gluteus maximus and the tensor fascia latae d. Vastus lateralis and vastus intermedius

C

The inguinal ligament is formed by the: a) Inferio-medial fusing of the fascia of the internal oblique and transverse abdominus muscles b) Deep fascia of the rectus abdominis mucle c) Rolling under of the edge of the aponeurosis of the external oblique d) Rolling under of the edge of the aponeurosis of the internal oblique

C

The landmark on the femur that provides an attachment for many of the muscles in the gluteal region is the: a. Intertrochanteric line b. Gluteal tuberosity c. Greater trochanter d. Quadrate tubercle

C

The lateral walls of the axilla (or axillary pyramid) is formed by: a) Pectoralis major and minor b) Thoracic wall and serratus anterior c) Intertubercular groove or bicipital groove of the humerus d) Latissimus dorsi and teres major

C

The least clinically significant form of spina bifida in which there is a bony defect, but no neural defect is called: a) Meningocele b) Meningomyelocele c) Spina bifida occulta d) Spina bifida cystica

C

The levator scapulae muscle: a) Assists with elevation of the scapula, superior rotation of the glenoid fossa b) Is innervated by the hypoglossal nerve c) Assists with elevation of the scapula and inferior rotation of the glenoid fossa d) Has an attachment on the medial third of the clavicle

C

The long head of the biceps femoris muscle: a. Is innervated by the common fibular (peroneal) division of the sciatic nerve and crosses only one joint b. Is innervated by the obturator division of the sciatic nerve and crosses only one joint c. Is innervated by the tibial division of the sciatic nerve and crosses two joints d. Is innervated by the deep fibular (peroneal) division of the sciatic nerve and crosses two joints

C

The medial and lateral pectoral nerves arise from the: a) Upper trunk b) Lateral cord c) Medial and lateral cord d) Medial cord

C

The median nerve receives contributions (proximally) from the: a) Posterior divison b) Long thoracic nerve c) Medial and lateral cords d) Posterior, lateral, and medial cords

C

The most common ankle sprain, putting stress on the ATF, CF, and PTF ligaments is: a) High ankle sprain b) Eversion ankle sprain c) Inversion ankle sprain d) Medial ligament sprains

C

The muscle NOT involved in medial rotation of the humerus is the: a) Pectoralis major muscle b) Subscapularis muscle c) Teres minor muscle d) Teres major muscle

C

The muscle that can extend, adduct, AND medially rotate the GH joint is the: a) Trapezius b) Posterior deltoid c) Latissimus dorsi d) Serratus anterior

C

The muscle that can extend, adduct, AND medially rotate the humerus at the glenohumeral joint is the: a) Trapezius b) Posterior deltoid c) Latissimus dorsi d) Serratus anterior

C

The muscle that does NOT receive at least partial innervation from the femoral nerve is: a. Pectineus b. Sartorius c. Gracilis d. Rectus femoris e. More than one of the above listed muscles does NOT receive innervation from the femoral nerve

C

The muscle that does NOT receive motor innervation from the dorsal rami of spinal nerves is? a) Spinalis b) Iliocostalis c) Trapezius d) Longissimus

C

The muscle that does not assist with MEDIAL rotation of the humerus at the GH joint is: a) Pectoralis major b) Latissimus dorsi c) Pectoralis minor d) None of the above

C

The muscle that does not help flex the wrist is the: a) Palmaris longus b) Flexor digitorum superficialis c) Pronator teres d) Flexor carpi radialis

C

The muscle that is NOT in the deep layer of spine musculature is the: a) Multifidus b) Rotatores c) Serratus posterior inferior d) Semispinalis thoracis

C

The muscle that is considered a key landmark of the anterior neck, dividing each side into anterior and posterior triangles is the: a) Digastric b) Omohyoid c) SCM d) Sternothyroid

C

The muscle that is considered the key landmark between the anterior and posterior triangles has the following actions of the head at the neck: a) NONE b) Contralateral lateral flexion and ipsilateral rotation c) Ispsilateral lateral flexion and contralateral rotation d) Bilateral contraction equalling extension

C

The muscle that will assist with the movement of rotation of the trunk to the left is the: a) Right internal oblique b) Right transverse abdominus c) Left internal oblique d) Left external oblique

C

The muscle that will assist with the movement of rotation of the trunk to the left is the: a) Right internal oblique b) Right transverse abdominus c) Left internal oblique d) Left external oblique

C

The muscles of the anterior abdominal wall may perform any of the following functions EXCEPT: a) Protection of the abdominal viscera b) Increase the intra-abdominal pressure c) Extend the trunk against resistance d) Aid in parturition (child birth)

C

The nerve supplying the muscles of mastication is: a. The facial nerve b. The mandibular branch of the facial nerve c. The mandibular division of the trigeminal nerve d. The maxillary division of the trigeminal nerve

C

The nerve that arises directly from the ventral rami of C5, C6, and C7 and innervates serratus anterior is the: a) Medial antebrachial cutaneous b) Suprascapular c) Long thoracic d) Phrenic

C

The nerve to the quadratus femoris also innervates the a. Piriformis b. Obturator internus c. Inferior gemellus d. Superior gemellus

C

The nerves crossing into the forearm anterior to the epicondyles of the humerus are the: a) Radial and ulnar b) Median and axillary c) Median and radial d) Median and ulnar

C

The origin of the Posterior Cerebral Artery is _______________ and it provides blood supply to the __________ region of the brain. a. Basilar artery / frontal lobes of the cerebral hemisphere b. Internal carotid artery / frontal lobes of the cerebral hemisphere c. Basilar artery / occipital lobe of cerebral hemisphere d. Internal carotid artery / occipital lobe of cerebral hemisphere

C

The pancreas is surrounded by the _________ caudally and the ___________ cephally. a. Duodenum, Stomach b. Liver, Spleen c. Spleen, Duodenum d. Stomach, Liver

C

The pectoralis major muscle has attachments on: a) The scapula, the humerus, and the sternum b) The clavicle, the scapula, and the sternum c) The clavicle, the humerus, and the sternum d) The clavicle, the humerus, and the xiphoid process

C

The projection of the scapula that extends laterally and anteriorly to form the acromion process is the: a) Coracoid process b) Lateral border c) Spine d) Notch

C

The rectus abdominus muscle is divided into two portions (left and right) by the: a) Linea semilunaris b) Conjoint tendon c) Linea alba d) Arcuate line

C

The rib that articulates with the junction of the sternum and the manubrium (sternal angle or angle of Louis) is the: a) 5th b) 3rd c) 2nd d) 1st

C

The right atrium: a) Receives blood from the oblique subclavian b) Is associated with the apex of the heart c) Contains the sinoatrial (SA) node d) Sends blood to the pulmonary trunk via the tricuspid valve

C

The sciatic nerve is made up of two main portions, which are the: a. Tibial and pudendal b. Femoral and popliteal c. Tibial and common peroneal d. Fibular and common peroneal

C

The spinal accessory nerve provides: a. Sensory innervation to the muscles of facial expression b. Motor and sensory innervation to the spinal nerves that form the brachial plexus c. Motor innervation to the sternocleidomastoid muscle d. Motor innervations to the rhomboid muscle

C

The statement that is TRUE regarding the longissimus thoracis muscle is: a) It is located in the thoracic region and has attachment on the spinous processes b) It is located in the lumbar region and has attachment on the transverse processes c) It is located in the thoracic region and has attachment on the transverse processes d) It is located in the lumbar region and has attachment on the spinous processes

C

The sternoclavicular joint: a) Is stabilized by the coracoclavicular ligament b) Is one of 4 bony articulations between the upper limb and the axial skeleton c) Is a saddle synovial joint with a fibrocartilaginous disc d) Allows very little motion

C

The structure that does NOT have an attachment on the coracoid process is the: a) Pectoralis minor muscle b) Coracoacromial ligament c) Long head of the biceps d) Coracobrachialis muscle

C

The structure that is formed by all of the posterior divisions of the brachial plexus is the: a) Posterior trunk b) Lateral cord c) Posterior cord d) Anterior trunk

C

The term "idiopathic scoliosis" means that: a) Treatment of the mother with iontophoresis during pregnancy is the cause of the scoliosis b) The cause of the scoliosis is a leg length discrepancy created by a lower extremity fracture c) The cause of the scoliosis is unknown d) The scoliosis was caused by an infection that occurred during the last hospital stay

C

The trochlea of the humerus: a) Lies lateral to the humeral capitulum b) Provides partial origin for the flexors of the forearm c) Articulates with the proximal end of the ulna d) Is located between the greater and lesser (humeral) tubercles

C

The typical vertebral column is made up of vertebrae in the following numbers: a) 8 cervical, 12 thoracic, 5 lumbar, 5 sacral, and 3-4 coccyx b) 7 cervical, 10 thoracic, 5 lumbar, 4 sacral, and 3-4 coccyx c) 7 cervical, 12 thoracic, 5 lumbar, 5 sacral, and 3-4 coccyx d) 8 cervical, 12 thoracic, 5 lumbar, 5 sacral, and 3-4 coccyx

C

The vertebral arch includes ALL of the following components: a) 2 TP, 2 pedicles, 2 lamina, and 1 SP b) 2 TPS and 2 lamina c) 2 pedicles, 2 lamina, that's it d) 2 pedicles, 2 lamina, 1 SP

C

The vertebral artery is fed by which of the following arteries? a. Brachiocephalic b. Left common carotid c. Subclavian d. Arch of the aorta

C

The very strong inverted 'Y' shaped ligament of the hip that is taut in full extension of the hip is the: a. Ischiofemoral ligament b. Sacrospinous ligament c. Iliofemoral ligament d. Pubofemoral ligament

C

There are times when different portions of a single muscle can have actions in opposite directions. An example is: a) Short and long heads of biceps b) Rhomboid major and minor c) Anterior and posterior deltoid d) Medial and lateral heads of triceps

C

This is characterized by synovial fluid that escapes from the knee joint and accumulate in the popliteal fossa a) Osgood-Schlatter Disease b) Condromalacia patella c) Baker's cyst d) Osteoarthritis

C

What is the military patch sign? a) Diminished sensation at pectoral major b) Pain at pectoral major in location of military patches c) Diminished sensation at lateral shoulder d) Bleeding in shape of a patch at the lateral shoulder

C

What is the primary action lost if the quadriceps femoris group is paralyzed? a. Extension of the hip b. Flexion of the knee c. Extension of the knee d. Flexion of the hip

C

What is the term for a decreased angle of inclination, caused by increased weight bearing in children, resulting in a shorter leg? a) Coxa valga b) Genu varum c) Coxa vara d) None of the above

C

What is the term for a valgus deformity shown with everted foot and where the longitudinal arches are decreased? a) Pes cavus b) Pes anserine c) Pes planus d) None of the above

C

What muscle makes the scary face? a) SCM b) Lagoon scary face c) Platysma d) Traps

C

Which muscles abducts digits away from midline? a) Lumbricals b) Palmar interossei c) Dorsal interossei d) Opponens pollicis

C

Which of the following does not have an attachment on the ASIS? a) Sartorius b) Inguinal ligament c) Pectineus d) TFL

C

Which of the following hamstring muscles is most lateral? a) Semitendinosus b) Vastus lateralis c) Biceps femoris d) Adductor magnus

C

Which of the following is incorrect pertaining to the rectus abdominis muscle or rectus sheath? a) The linea alba separates (lies in the midline between) the two rectus muscles b) The external oblique aponeurosis contributes to the anterior wall of the sheath c) The posterior layer of the sheath is composed of the apoenuroses of the internal oblique and the transversalis fascia throughout the extent of the sheath d) The tendinous insertions/intersections (between the muscle and the anterior later of sheath) account for the abdominal 6 pack when muscular individuals tense the abS

C

Which of the following is incorrect pertaining to the ribs? a) Ribs 3-10 are typical b) Ribs 11 and 12 are typically floating ribs c) The tubercle of a typical rib attaches to the inferior articular facet of the corresponding vertebrae d) The head of a typical rib articulate with the bodies of two vertebrae

C

Which of the following is not an origin of the tibialis anterior? a) Superolateral surface of tibia b) Interosseous membrane c) Lateral surface of fibula d) Lateral condyle of tibia

C

Which of the following muscles typically receive a dual motor innervation? a. Adductor Longus b. Adductor Brevis c. Adductor Magnus d. Gracilis e. More than one of the above receives a dual motor innervation

C

Which of the following statements is FALSE as related to the cribriform plate? a. It is part of the ethmoid bone. b. It possesses numerous tiny foramina that transmit olfactory nerves. c. It is located in the occipital cranial fossa. d. It lies adjacent to the crista galli.

C

Which of the following statements is true for the sacrum? a) It is convex anteriorly b) Auricular surfaces (lateral roughened) articulate with lumbar vertebrae c) The sacral canal is the extension of the vertebral canal d) It consists of seven fused vertebrae

C

While coaching intramural softball Professor Robbins inured his middle finger. He knows that a disruption in the terminal extensor tendon means he is currently unable to: a) Flex the DIP joint of the injured finger b) Flex the PIP joint of the injured finger c) Extend the DIP joint of the injured finger d) Extend the PIP joint of the injured finger

C

Why is the Right Renal Artery longer? a. The inferior Vena Cava is located on the Left b. The descending Aorta in on the right side of the spinal column c. The descending Aorta is on the left side of spinal column d. The Right Renal Artery is not any longer than the Left

C

With a patient in the anatomical position, the thumb is ___ to the median plane: a) Dorsal b) Medial c) Lateral d) Ventral

C

With structural scoliosis one might expect that: a) 60-80% of cases are found in males b) It can be corrected by having the patient lie down c) 80% of cases are of unknown etiology (idiopathic) d) The curvature is named for the direction of the concavity laterally

C

You observe Dr. Freund perform a muscle test by having the patient abduct her arm 90 degrees and horizontally adduct the arm against resistance. What muscle (or part of a muscle) is she testing? a) Upper trapezius b) Rhomboid major c) Pectoralis major d) Pectoralis minor e) Serratus posterior superior

C

A 27 y.o. male patient you are working with was involved in a serious accident in which the anterior longitudinal ligament was completely torn. As a consequence, the patient is likely to demonstrate an abnormally increased movement in: a) Flexion of the trunk b) Trunk rotation to the right c) Trunk rotation to the left d) Extension of the trunk

D

A person with an injury to the axillary nerve is likely to exhibit weakness in the following movements: a) Upward rotation of the scapula b) Extension at the elbow joint c) Adduction and internal rotation of the glenohumeral joint d) Abduction and external rotation of the glenohumeral joint

D

A simple mastectomy for breast cancer involves removal of: a) All breast tissue and underlying muscles b) Only one quadrant of breast tissue c) All of the lymph nodes that drain the breast d) All breast tissue superficial to the retromammary space

D

A torn ACL will NOT result in unwanted movement in which direction? a) Posterior displacement of the femur b) Anterior displacement of the tibia c) Hyperextension d) Hyperflexion

D

According to Moore: Piriformis is a: a. Femoral head stabilizer in acetabulum b. Hip abductor when thigh is flexed c. Lateral rotator d. More than one of the above is correct

D

According to Moore: all of the following muscles attach the pectoral or shoulder girdle to the axial skeleton EXCEPT: a) Trapezius b) Levator scapulae c) Latissimus dorsi d) Serratus posterior superior

D

After several years of successful practice as a DPT, hand therapist, Mackenzie continues on to medical school becoming a hand surgeon. She sees a patient named Nathan who had cardiac surgery two months ago. The patient is left handed and since his surgery he complains of functional loss in his left hand. He has a weak KEY PINCH (lateral pinch) reports he is having problems holding the key to unlock his front door. From just the patient complaints, Dr. Borrowman believes that Nathan has had an injury to: a) The musculocutaneous nerve b) The median nerve c) The axillary nerve d) The ulnar nerve

D

Anterior ramus supply nerve fibers to: a) The skin b) The muscles, joints and skin of the back of the body c) The CN's d) The muscles, joints, and skin of the front of the body

D

Blood supply from the aorta to the brain on the right side of the thorax is as follows: a) Common carotid, internal carotid b) Brachiocephalic, internal and external carotid, common carotid c) Internal and external carotid, common carotid d) Brachiocephalic, common carotid, internal carotid

D

Both heads of the biceps femoris have their distal attachments on a. The medial side of the proximal tibia b. The lateral side of the proximal femur c. Medial side of the head of the fibula d. Lateral side of the head of the fibula

D

Characteristics of the spine include: a) 33 vertebrae b) Cervical, thoracic, lumbar, sacral regions c) Included in axial portion of the skeleton d) All of the above

D

Cubital Tunnel syndrome typically involves which of the following nerves? a) Radial b) Median c) Axillary d) Ulnar e) Musculocutaneous

D

Cutaneous innervation to the skin of the dorsum (posterior surface) of the hand overlying metacarpals 1 & 2 is typically derived from which of the following nerves? a) Ulnar b) Median c) Axillary d) Radial

D

Digestive enzymes and bile enter into the duodenum from: a. Common bile duct b. Pyloric duct c. Cardiac duct d. Ampulla of Vater or hepatopancreatic duct

D

Dr. Cope is carrying a scalpel in her back right pocket when she leans against the counter in the anatomy lab. Dr. Zimmerman, suspects a laceration to the superior gluteal nerve. Thus she performs the following test to assess for this damage: a. Tortolinni b. Ober c. Ortolonni d. Trendelenberg

D

Dr. Johansson had a traumatic workplace injury to just the median nerve at the mid-forearm level, with complete laceration of the nerve and subsequent loss of function. Hannah Cunningham, DPTS tells her that the muscle of the thumb that would continue to have full innervation and function is the: a) Opponens pollicis b) Abductor pollicis brevis c) Flexor pollicis brevis superficial head d) Adductor pollicis

D

Dr. Palermo is assessing hip flexor tightness. In proper testing position he observes that the patient's thigh is elevated off of the table and that the knee is extended. Dr. Palermo correctly believes there to be tightness of the: a. Semitendinosus b. Vastus intermedius c. Biceps femoris d. Rectus femoris

D

Dr. Stetts sees a patient who reports paresthiesia of digit III pad (middle finger), you know that his is a dermatomal pattern for: a) C4 b) C5 c) C6 d) C7 e) C8

D

Dr. Turcotte tells us that she is treating a patient who has a slipped epiphysis of the femoral head. This means that her patient: a. Is most likely a subadult b. Has participated in activities that created a shearing force at the fusion line of the femoral head c. May have coxa vara d. More than one of the above is true

D

Dr. Zimmerman was walking through the swamps of rural Mebane, tracking a rare Pilleated woodpecker when she stumbled over a submerged stump. Dr. Cope, her hiking partner quickly came upon the scene and assessed vital signs, including palpation of the radial pulse, located: a) At the distal end of the humerus, medial to the common extensor attachment b) At the distal end of the radius, medial to the median nerve c) At the distal end of the humerus, medial to the tendon of the biceps brachii d) At the distal end of the radius, lateral to the tendon of the flexor carpi radialis

D

From lateral to medial, respectively, the landmarks of the pelvis are the: a) Symphysis pubis, pubic crest, ASIS b) Symphysis pubis, pubic tubercle, ischial spine c) Pubic tubercle, ischial spine, symphysis pubis d) ASIS, pubic tubercle, symphysis pubis

D

From superior to inferior the landmarks of the anterior neck are: a) Thyroid cartilage, laryngeal prominence, supraclavicular fossa b) Thyroid cartilage, hyoid bone, jugular notch c) Laryngeal prominence, thyroid cartilage, sternal notch, supraclavicular fossa d) Hyoid bone, laryngeal prominence, jugular notch

D

Function(s) of the lymphatic system include: a) To protect the body through immune response b) To transport fluid from the interstitial spaces to the bloodstream c) To help distribute hormones, nutrients, and wastes d) All of the above

D

Functions of the liver include: a. Breakdown of fat b. Storage c. Detoxification d. More than one of the above is correct

D

Haley, DPTS, is working with a 14 year old patient who sustained several fractures around the elbow following from a fall from a horse. She must asses the patient's range of motion at the elbow. Rebecca recalls from anatomy class that the olecranon fossa of the humerus is filled with the olecranon process of which of the following? a) The radius with the elbow in extension b) The ulna with the elbow in flexion c) The radius with the elbow in flexion d) The ulna with the elbow in extension

D

Identify the TRUE statement(s) about bursa: a. They decrease friction between tendons and bones b. They assist in lubricating synovial joints c. They are fluid filled sacs d. All of the above ARE TRUE about bursa

D

If a DPT student stuck her tongue out, which CN is she demonstrating integrity of? a) IX b) X c) XI d) XII

D

Kyphosis in an older person is can be related to: a) Osteoarthritis b) Osteoporosis c) Disc degeneration d) All of the above can increase the likelihood of developing kyphosis of the spine

D

Lightly touch lateral arm over deltoid: Use the following list to match dermatomal and myotomal patterns. a) C2 b) C3 c) C4 d) C5 e) C6

D

Lindsey, DPTS, is working with a patient who has experienced an injury resulting in a residual weakness of the biceps brachii muscle. Of the muscles below, the only one that can compensate for the weakness in elbow flexion is the: a) Anterior deltoid b) Anconeus c) Coracobrachialis d) Brachialis

D

Match the Facial Action with the muscle that is the prime mover: Frowning a. Risorius b. Procerus c. Nasalis d. Depressor anguli oris e. Zygomatic major and minor

D

Metacarpophalangeal joints of digits II - V are comprised of the: a) Heads of distal phalanges and bases of the metacarpals b) Bases of distal phalanges and heads of the metacarpals c) Heads of proximal phalanges and bases of the metacarpals d) Bases of proximal phalanges and heads of metacarpals

D

Natalia says the ductus arteriosis has the following properties. a) Becomes the Ligamentum Arteriosis after birth b) Shunts oxygenated blood during prenatal life c) Provides a pathway from pulmonary trunk to the aorta in prenatal life d) All of the above are correct

D

Nick tells us that the "Carpal Tunnel" contains a) Tendons of Extensor Digitorum b) Tendon of Adductor Pollicis c) Tendon of Abductor Pollicis Longus d) Median Nerve e) All of the above are correct

D

Normal Curves of the spine include: a) Anterior convexity at cervical; posterior convexity lumbar and thoracic b) Anterior concavity cervical,lumbar; posterior concavity thoracic c) Anterior convexity cervical, thoracic; posterior convexity lumbar d) Posterior concavity cervical and lumbar; anterior concavity thoracic

D

On a typical lumbar vertebra, the portion that connects the body to each transverse process is known as the: a) Lamina b) Lateral longitudinal ligament c) Zygapopohyseal (facet) joint d) Pedicle

D

Osteochondrosis or inflammation of the tibial tuberosity, a condition associated with running and jumping and which gives adolescents painful knees, is more commonly known as: a. Legg-Calve-Perthes disease b. Slipped capital femoral epiphysis c. Baker's cyst d. Osgood-Schlatter's disease

D

RESIST THE MVMT: Abduct arm at glenohumeral joint: Use the following list to match dermatomal and myotomal patterns. a) C2 b) C3 c) C4 d) C5 e) C6

D

Research tells us that the more effective method of working the abdominal muscles is to: a) Perform toe touches by flexing forward in a standing position b) Sit all day in a bean bag chair c) Eat nachos and watch football d) Planking crunches are safer for the spine than traditional sit ups

D

Retroversion is when: a. The angle of torsion of the neck of the femur is increased and the toes turn in with head alignment b. The angle of torsion of the neck of the femur is increased and the toes turn out with head alignment c. The angle of torsion of the neck of the femur is decreased and the toes turn in with head alignment d. The angle of torsion of the neck of the femur is decreased and the toes turn out with head alignment

D

Roles of the integumentary system include: a) Protection from environment b) Heat regulation c) Synthesis and storage of Vitamin D d) All of the above e) A&B

D

Sarah Hynes, DPT is assisting in the emergency room when the attending physician examines a patient holding the proximal interphalangeal joint of the little finger and asking the patient to flex the distal phalanx. This maneuver is testing the integrity of which muscle at which joint? a) FDP at MCP b) Lumbrical at MCP c) Lumbrical at DIP d) FDP at DIP

D

Sartorius and quadratus femoris have _________ in common a. Abduction of the femur at hip b. Flexion of the femur at the hip c. Innervation d. Lateral rotation of the femur at the hip

D

Shoulder pain would NOT be present with: a) The onset of a heart attack b) An impingement of the supraspinatus c) Systemic disease involving the breast d) A Neuromuscular condition of the pelvis

D

The Cribiform Plate: a. Is a portion of the Ethmoid bone b. Provides a safe place for the olfactory bulbs of CN I to lie in c. Lies lateral to the Crista Galli d. More than one of the above are correct

D

The Inion/bump of knowlege is observed on what bone? a) Sternum b) Clavicle c) Scapula d) Occipital bone of cranium e) Mandible

D

The Thomas test can be used to evaluate tightness or contracture of which of the following muscles? a. Iliacus b. Psoas major c. Rectus femoris d. All of the above can be tested for tightness with the Thomas test

D

The Ulnar Nerve innervates: a) Lumbrical to digit #3 b) 2nd Dorsal Interosseous c) 2nd Palmar Interosseous d) More than one of the above is innervated by the ulnar nerve

D

The anterior divisions of the brachial plexus contribute to create all of the following terminal branches EXCEPT: a) Musculocutaneous nerve b) Median nerve c) Ulnar nerve d) Radial nerve

D

The cords of the brachial plexus are named for: a) Their relationship to the clavicle b) Their relationship to the pectoralis minor tendon c) Their relationship to the subclavius muscle d) Their relationship to the axillary artery

D

The distal attachment of the long head of the triceps brachii is the: a) Infraglenoid tubercle of the scapula b) Supraglenoid tubercle of the scapula c) Distal end of the ulna at the olecranon d) Proximal end of the ulna at the olecranon

D

The head of the ULNA articulates with: a)The Triquetrum and Lunate b) Triquetrum only c) The Lunate and Scaphoid d) Ulnar notch of the distal radius

D

The laminae of a vertebra join together posteriorly to form the: a) Pedicle b) Transverse process c) Intervertebral foramen d) Spinous process

D

The ligament that connects the laminae of adjacent vertebrae is the: a) Posterior longitudinal ligament b) Laminal ligament c) Ligamentum nuchae d) Ligamentum flavum

D

The ligament that connects the most posterior part of the spinous processes of adjacent vertebrae is the: a) Posterior longitudinal ligament b) Infraspinous ligament c) Ligamentum nuchae d) Supraspinous ligament

D

The lungs : a) Right has 3 lobes, Left has 2 lobes b) Right has 2 lobes, Left has 3 lobes c) Oblique fissure separates upper and lower lobes d) More than one of the above is TRUE

D

The mastoid notch of the mastoid process of the temporal bone is the superior attachment for the: a) Rectus abdominus muscle b) Anterior portion of pectoralis major muscle c) Pectoralis minor muscle d) SCM muscle

D

The meninges that surround the brain, providing protection and support include from outer to inner most: a. Pia, Dura, Arachnoid b. Pia, Arachnoid, Dura c. Dura, Pia, Arachnoid d. Dura, Arachnoid, Pia

D

The most common cause of thoracic outlet syndrome is: a) Twisting or torsion b) Laceration c) Infection d) Compression

D

The most likely reason for avascular necrosis in an 80-year-old woman following hip fracture is the: a. Fracture included the head of the femur. b. Fracture split the neck of the femur longitudinally. c. Fracture included lacerating the transverse acetabular ligament. d. Branches of the medial femoral circumflex artery were severed by the fracture

D

The muscle more well developed in people who participate in activities such as rock climbing, gymnastics, and kayaking and that has its superior attachment on the inferior surface of the middle third of the clavicle is the: a) External intercostal muscle b) Serratus anterior muscle c) Pectorialis minor muscle d) Subclavius muscle

D

The muscle that attaches to the costal cartilage of ribs 5-7 and the pubic bone is the: a) Transversus abdominis b) Quadratus lumborum c) Serratus inferior d) Rectus abdominis

D

The muscle that flexes the thumb at the interphalangeal joint is the: a) Flexor pollicis brevis b) Flexor digitorum profundus c) Flexor digitorum superficialis d) Flexor pollicis longus

D

The muscle that is not contracting while writing, as you are in taking this exam is: a) Adductor pollicis b) Flexor pollicis longus FPL c) Lumbrical d) Flexor carpi radialis longus

D

The pancreatic duct joins with the ____ and then empties into the ______: a. Gallbladder, duodenum b. Common bile duct, jejunum c. Gallbladder, jejunum d. Common bile duct, duodenum

D

The part of the femur that is most susceptible to fracture in conjunction with osteoporosis or avascular necrosis of the femur is: a. Mid-shaft b. Medial condyle c. Along the intertrochanteric line d. Neck e. Intercondylar area

D

The photograph above depicts a woman with a lesion to a cranial nerve. She is being asked to look straight ahead with both eyes. The muscle that is not working properly is? a. Superior rectus b. Medial rectus c. Inferior rectus d. Lateral rectus e. Superior oblique

D

The pituitary gland is attached to the inferior aspect of the brain and sits in the ____ of the inner aspect of the cranium. a. Falx Cerebri b. Crista galli c. Corpus Callosum d. Sella turcica

D

The primary curves of the spine are called such because they: a) Are functionally of greater importance than the cervical and lumbar curves b) Are located more superior on the vertebral column than the secondary curves c) Have larger curvatures than the secondary curves d) Develop during the fetal period

D

The pterion: a. Is posterior-inferior to the external auditory meatus b. Is another name for orbitomeatal line c. Makes up the junction at the occipital, parietal and temporal bones d. H-formation of sutures at the junction of the temporal, parietal, and frontal bones

D

The statement that is FALSE concerning the shoulder joint is: a) The glenohumeral to scapulothoracic movement ratio is 2:1 b) The strength of the joint depends largely upon the strength of the surrounding musculature c) Dislocation of the shoulder joint is common d) The axillary artery lies posterior to the shoulder joint

D

The statement that is TRUE concerning the rotator cuff (RTC) is: a) It adds stability to the shoulder joint b) It is formed by the tendons and short muscles around the shoulder joint c) The distal attachments are fused together with the capsule surrounding the shoulder joint d) All of the above are true about the RTC

D

The statement that is false about vertebrae is: a) Cervical vertebrae all have openings in the transverse processes b) The thoracic vertebrae have costal facets c) The lumbar vertebrae have relatively massive bodies d) There are eight cervical vertebrae

D

The structure passing through the inguinal canal in males is the: a) Round ligament of the uterus b) Inguinal ligament c) Urethra d) Spermatic cord

D

The superior orbital fissure transmits the following: a. CN III b. Trochlear c. CN V 1 d. More than one of the above is transmitted through the superior orbital fissure

D

The superior thoracic aperture (opening to the thorax) is also known as the: a) Thoracic outlet syndrome b) Clavicles c) Pectoral girdle d) Thoracic inlet

D

The tensor fasciae latae assists with the following action(s): a. Hip flexion and medial rotation b. Maintain knee extension c. Hip abduction d. TFL assists with all of the above actions

D

The vertebral arteries pass through what structures on vertebrae C1-C6? a) Intervertebral foramina b) Vertebral foramina c) Circulatory foramina d) Transverse foramina

D

The vertebral artery supplies the area of the brain where it enters the cranium. This would be which portion of the brain? a. Temporal Lobe b. Parietal Lobe c. Frontal Lobe d. Cerebellum

D

Things that support digestion of food include: a. Teeth b. Saliva c. Cooking d. All of the above

D

What is the purpose of the uvula? a. Allows taste to the posterior 1/3 of the tongue b. Prevents nasogastric regurgitation c. Gag reflex d. B and C e. All of the above

D

What movement is least limited in the thoracic region? a) Extension b) Lateral flexion c) Abduction d) Rotation

D

When a vertebral disc herniates, it will have the most significant clinical implications when it herniates: a) Anterolaterally b) Medially c) Superiorly d) Posterolaterally

D

Which ligament of the knee prevents valgus and external rotation? a) LCL b) ACL c) PCL d) MCL

D

Which muscle originates on the lateral condyle and the popliteal surface of the femur? a) Soleus b) Popliteus c) Plantaris d) Gastroc

D

Which of the following carpal bones is typically observed in the "proximal" row? a) Trapezium b) Capitate c) Hamate d) Triquetrum

D

Which of the following cranial nerves contain both motor and sensory fibers? a. Olfactory, Optic and Hypoglossal b. Vestibulocochlear, Spinal Accessory and Hypoglossal c. Optic, Oculomotor, and Trochlear d. Trigeminal, Facial and Vagus

D

Which of the following is not a sacroiliac ligament? a) Anterior b) Interosseus c) Posterior d) All of the above are sacroiliac ligaments

D

Which of the following items is not contained within the femoral triangle? a. Femoral artery b. Pectineus c. Femoral nerve d. Adductor magnus muscle

D

Which of the following muscles cross the knee joint? a. Gluteus medius b. Biceps femoris c. Rectus femoris d. More than one of the above muscles cross the knee joint

D

Which of the following muscles does NOT assist in flexion of the femur at the hip? a. Sartorius b. Tensor fasciae lata c. Illiacus d. Psoas minor

D

Which of the following muscles does not attach to pes anserine? a) Semitendinosus b) Gracilis c) Sartorius d) Gastroc

D

Which of the following muscles is NOT in the anterior compartment of the leg? a) Fibularis tertius b) Tibialis anterior c) Extensor digitorum longus d) Flexor digitorum longus

D

Which of the following special tests is a test for congenital dislocation of the hip? a) Ober's Test b) Ely Test c) Trendelenburg Test d) Ortolani's Test

D

Which of the following statements about the temporomandibular joint is FALSE? a) It is a hinge type synovial joint b) It contains an articular disc c) The movements at this joint consist of elevation/depression, protrusion/retrusion, and lateral movement d) The muscles that move this joint are innervated by the facial nerve

D

Which of the following statements is TRUE as related to the deltoid muscle? a) The anterior portion and the short head of the biceps brachii assist with glenohumeral extension b) The middle portion and the supraspinatus assist with glenohumeral adduction c) The posterior portion and the long head of the triceps assist with glenohumeral flexion d) It is innervated by the axillary nerve e) More than one of the above is TRUE

D

Which of the following statements is TRUE of the bones of the lower extremities? a. The femur is the longest b. The femur is the strongest c. The femur is the heaviest d. More than one of the above is correct

D

Which of the following statements is TRUE pertaining to the bony pelvis? a. The pelvic girdle is composed of only the right and left innominate bones b. The pubic symphysis provides stress relief posteriorly c. The ala of the ilium helps to form the acetabulum d. The ischial spine separates the greater and lesser sciatic notches

D

Which of the following structures course through the Optic Foramen (canal) of the orbit to form the optic chiasm? a. CN III b. CN IV c. CN VI d. CN II

D

Which of the following structures maintains constant tension on the cusps of the atrioventricular (AV) valves? a) Trabecular Carnae b) Pectinate muscles c) Annulus fibrosus d) Chordate tendinae

D

While on a neurology rotation, the attending physician asks you to test a patient's femoral nerve function. Which of the following would you do? a. Have the patient do a sit-up while you hold and resist his thigh b. Have the patient rotate his thigh at the hip medially while you resist this movement c. Have the patient extend his thigh at the hip against your resistance d. Have the patient extend his leg at the knee against your resistance e. Have the patient flex his leg at the knee against your resistance

D

While on his clinical affiliation, Nate reported treating a client who after returning from Alaska and is now unable to whistle, play the flute, or wink because of sudden loss of control of certain muscles on one side of his face. What is the mostly likely diagnosis? a. TMJ joint derangement b. Facial neuralgia c. Herpes zoster ophthalmicus d. Bell's palsy

D

You ask a patient to protrude their tongue out. You note the tongue deviating to the right side. What cranial nerve and which side is affected? a. IX; Right b. IX: Left c. XII; Left d. XII; Right

D

Your 25-year-old patient, Kelly, arrives for Physical Therapy, she states has not felt well the entire school day. She complains of severe right lower quadrant pain. The area is tender to the touch. The patient displays antalgic (avoiding pain position) posture, guarding her abdomen with slouched posture. She is unable to find a position of comfort. She denies any traumatic injury since you saw her (3) days ago. What is the most likely the cause of her pain? a. Ruptured spleen b. Stomach Ulcer c. Ectopic Pregnancy d. Appendicitis e. None of the above

D

Your patient reports that she has numbness along the ulnar side of her hand and ring and little finger of her right hand. You remember that the dermatomal pattern she is describing is: a) C6 b) C5, 6, 7 c) C5 d) C8

D

A patient with a diagnosis of spondylosis would typically have: a) Widening of the vertebral canal b) Osteoarthritic changes in the vertebral bodies and facets c) A lesion or defect of the pars interarticularis in the lumbar region d) A disease with the patient at risk for trunk flexion contractures and reduced rib mobility e) More than one of the above statements is true about spondylosis

E

A patient with superior oblique issues is likely to report problems with the following a) Walking downstairs b) Talking c) Reading a book that is resting in their lap d) A and B e) A and C

E

Bell's Palsy involves which of the following cranial nerves? a) III b) V c) II d) IV e) None of the above are correct

E

Biceps Femoris and Sartorius both: a. Are innervated by the tibial nerve b. Flex the leg at the knee c. Extend the leg at the knee d. Externally rotate the femur at the hip e. Flex the leg at the knee and externally rotate the thigh at hip

E

Colby says the following structures are contained within the hilum of the lung? a) Pulmonary Artery b) Pulmonary Veins c) Lymph d) Bronchus e) All of the above are contained within the hilum of the lung

E

Diplo (bilateral occurrence) occurs in the cranial bones of the following areas: a. Mandible b. Maxilla c. Temporal d. Parietal e. More than one area above has a bilateral occurrence of bones

E

Levator Palpabrea Superioris: a. Elevates the eyelid and is innervated by the Abducens nerve b. Elevates the eyeball and is innervated by the Trochlear nerve c. Elevates the eyeball and is innervated by the Abducens nerve d. Elevates the eyelid and is innervated by the Trochlear nerve e. None of the above are correct

E

Maggie says: "Components of the Apley Scratch Test Include assessing": a) External rotation of the humerus at the GHJ b) Internal rotation of the humerus at the GHJ c) Extension of the humerus at the GHJ d) Flexion of the humerus at the GHJ e) The Apley Scratch Test includes assessing all of the above movements at the GHJ

E

Match the Facial Action with the muscle that is the prime mover: Smiling (corners of mouth pulled superiorly) a. Risorius b. Procerus c. Nasalis d. Depressor anguli oris e. Zygomatic major and minor

E

Ptosis of the upper eye lid would most probably result from a lesion in which of the following cranial nerves? a. VII b. VIII c. V d. VI e. III

E

The Sternoclavicular joint: a) Is the only site of articulation between the axial & appendicular skeletons b) Is a saddle joint c) Contains a fibrocartilaginous disc d) Commonly dislocates e) More than one of the above are true

E

The Vagus Nerve: a. Provides motor innervation to Buccinator for whistling b. Carries taste from the posterior 1/3 of the tongue to the Central Nervous System c. Carries NO parasympathetic fibers to many of the organs d. Provides NO sensory innervation to the larynx e. None of the above are correct

E

The cubital fossa: a) Is bounded (in part) by the Pronator teres muscle b) Contains the tendon of the Biceps muscle c) Contains the Brachial artery d) Contains the median cubital vein e) More than one of the above are correct

E

The muscle that does not have an attachment on the posterior aspect of the femur is the: a. Vastus lateralis b. Rectus femoris c. Vastus intermedius d. Vastus medialis e. More than one of the above is correct

E

Which of the following associations is incorrect? a) Saddle joint - carpometacarpal joint of the thumb b) Condyloid joint - metacarpal phalangeal joint (big knuckles) c) Ball and socket joint - hip joint d) Plane joint - AC joint e) Pivot joint - radiocarpal joint

E

Which of the following carpal bones is typically observed in the "distal" row? a) Trapezium b) Capitate c) Hamate d) Triquetrum e) More than one of the above are located in the distal row

E

Which of the following does not go through the greater sciatic notch? a) Inferior gluteal nerve b) Sciatic nerve c) Piriformis d) Superior gluteal artery e) All of the above go through the greater sciatic notch

E

Which of the following is not associated in the 5 Ps of compartment syndrome? a) Pallor b) Paresis c) Pulselessness d) Pain e) All of the above are associated with compartment syndrom

E

Which of the following muscles extend the leg at the knee? a. Rectus Femoris b. Vastus Lateralis c. Gracilis d. Vastus Medialis e. More than one of the above extend the leg at the knee

E

Which of the following muscles is NOT a dorsiflexor of the foot? a) Tibialis anterior b) Extensor hallucis longus c) Fibularis tertius d) Extensor digitorum longus e) All of the above are dorsiflexors of the foot

E

Which of the following muscles is NOT a plantarflexor of the foot? a) Tibialis posterior b) Flexor hallucis longus c) Flexor digitorum longus d) Gastroc E) All of the above are plantarflexors of the foot

E

Which of the following muscles is NOT in one of the posterior compartments of the leg? a) Soleus b) Tibialis posterior c) Fibularis brevis d) Fibularis brevis e) More than one of the above are not in the posterior compartment of the leg

E

Which of the following muscles originate from the ischial tuberosity? a. Short head of biceps femoris b. Iliopsoas c. Semimembranosus d. Long head of biceps femoris e. More than one of the above originates on the ischial tuberosity

E

Which of the following statements apply to BOTH Serratus Anterior AND Pectoralis Minor? a) They both attach (at least in part) on Ribs 3 through 5 b) They both act upon the humerus (directly) c) They both receive innervation via some combination of the C5 through T1 spinal nerves d) They both are innervated by the Musculocutaneous Nerve e) More than one of the above is correct

E

Which of the following statements is TRUE about the vertebral arteries? a. They arise from the subclavian artery b. They ascend through transverse foramina of cervical vertebrae c. Enter the cranial cavity through the foramen magnum d. Join together to form the basilar artery e. All of the above are true statements about vertebral arteries

E

Which of the following statements is correct pertaining to the extensor carpi radialis longus and brevis muscles? a) When only the two muscles act together, wrist extension and abduction is produced b) When the two muscles act in concert with extensor carpi ulnaris wrist extension is produced c) When the two muscles act in concert with flexor carpi radialis abduction is produced d) One of the muscles is innervated by the ulnar nerve and the other by the radial nerve. e) More than one of the above answers is correct

E

Which of the following statements pertaining to the principle extensor of the forearm at the elbow is TRUE? a) It inserts upon the greater tubercle of the humerus b) It's tendon is the most commonly torn portion of the Rotator Cuff c) It originates (in part) from the scapula d) It is innervated by the Radial nerve e) More than one of the above is correct

E

Which of the following statements pertaining to the principle lateral rotator of the humerus is TRUE? a) It is innervated by the lower subscapular nerve b) It inserts upon the lesser tubercle of the humerus c) It does NOT contribute to formation of the Rotator Cuff d) It's tendon is the most commonly torn portion of the Rotator Cuff e) It does NOT originate on the vertebrae

E

Extensor digitorum longus has what action?

Extend lateral four digits

Which structure is used to designate which cruciate ligament is anterior and which is posterior?

Intercondylar eminence of the tibia

Due to its strength and the unlikely trauma occurring in the medial to lateral direction, this ligament is uncommonly torn

LCL

This collateral ligament inserts on its underlying meniscus

MCL

Which three structures are often referred to as the "Unhappy triad"?

MCL, ACL, medial meniscus

Which knee ligament prevents hyperflexion?

PCL

Fibularis longus and brevis are innervated by which nerve?

Superficial fibular

The flexors of the toes are innervated by which nerve?

Tibial


Ensembles d'études connexes

Blood-Brain Barrier, Cerebrospinal Fluid and Ventricles

View Set

culture diversity midterm chapters 1-5

View Set

Geology Chapter 17 Glencoe Online Science Test Assessment

View Set

2101 Final - Legal Principals in Nursing

View Set